Re: [obm-l] FÍSICA CONCEITUAL!

2005-10-20 Por tôpico Rogerio Ponce
Bom dia Jorge e colegas da lista!

Acabei de acordar, mas ja' estou PASMADO com o
comentario abaixo!

 --- Jorge Luis Rodrigues e Silva Luis escreveu: 
> 
> PASMEM! Todos, pois com relação à barcaça de rio,
> carregada com cascalho, me 
> parece que a resposta foi precipitada diante da
> maior "pegadinha física" da 
> história da ciência. 



Se eu nao perdi alguma coisa, a unica resposta na
lista foi dada corretamente pelo Claudio, e acompanha
o senso comum: basta adicionar cascalho 'a barcaca
para que ela passe por baixo da ponte.


Relembrando, o enunciado original era:
"Uma barcaça de rio, carregada com cascalho,
aproxima-se de uma ponte baixa, sob a qual não pode
passar. Dever-se-ia remover ou adicionar cascalho à
barcaça?"


Dessa forma, pelo fato de estarmos num rio, a adicao
ou subtracao de cascalho 'a barca ou ao rio nao altera
o nivel da agua no rio. Basta adicionar cascalho 'a
barcaca para que ela passe sob a ponte. Ponto p/ o
Claudio.

Mas, mesmo que estivessemos em um "canal de navegacao"
fechado (nenhum exagero imaginarmos que a densidade
relativa do cascalho seja 1.6, e que tal canal tenha
pelo menos 5 vezes o comprimento da barca, certo?),
ainda assim, a adicao de cascalho 'a barcaca seria a
solucao correta: 

Quando a pilha de cascalho aumenta de 1m , o casco
afunda de 1.6m , e o nivel da agua no canal sobe de
1.6m/5 (ou menos), ou seja, o topo da pilha desce
28cm. E nem precisamos supor que o cascalho seria
retirado do fundo do canal.

Nao entendi que pegadinha foi essa.

[]'s
Rogerio Ponce.









___ 
Promoção Yahoo! Acesso Grátis: a cada hora navegada você acumula cupons e 
concorre a mais de 500 prêmios! Participe! http://yahoo.fbiz.com.br/
=
Instruções para entrar na lista, sair da lista e usar a lista em
http://www.mat.puc-rio.br/~nicolau/olimp/obm-l.html
=


Re: [obm-l] Probabilidade

2005-10-17 Por tôpico Rogerio Ponce
Oi Adroaldo,
o Nicolau ja' deu a resposta incluindo alguns links
que vc pode (e deve) examinar.
Entretanto, nao custa salientar que, se antes da
abertura de porta, a probabilidade de ganhar era de
1/3, entao de cada 3 vezes que vc vai ao programa, em
2 vezes vc comeca com um monstro na sua porta. E nao
importa o que o apresentador faca, o monstro continua
la'.
Portanto, se vc nao trocar de porta, vc ganhara' um
carro em somente 1/3 das vezes, em vez de 50% .
A continuacao do raciocinio vc pode elaborar...
[]s
Rogerio Ponce.


--- Adroaldo Munhoz <[EMAIL PROTECTED]> escreveu:
nao vale a pena trocar de porta. antes a probabilidade
de ganhar era 1/3 e, apos abrir a porta, passou a
ser1/2, ou seja, 50% de o monstro estar na porta
escolhida e 50% de estar na outra.








___ 
Promoção Yahoo! Acesso Grátis: a cada hora navegada você acumula cupons e 
concorre a mais de 500 prêmios! Participe! http://yahoo.fbiz.com.br/
=
Instruções para entrar na lista, sair da lista e usar a lista em
http://www.mat.puc-rio.br/~nicolau/olimp/obm-l.html
=


Re: [obm-l] ESTRATÉGIA VENCEDORA!

2005-10-03 Por tôpico Rogerio Ponce
Olá Chicao e Johann,
parece-me que um jogador pode tirar pedras de qualquer
pilha, e que a estratégia é tentar sempre deixar as
pilhas com mesmo número de pedras.
Assim, se ninguém vacilar, o segundo jogador sempre
ganha: basta "repetir" a jogada do primeiro,
invertendo a pilha escolhida.
[]'s
Rogerio Ponce.


--- Chicao Valadares <[EMAIL PROTECTED]>
escreveu:

> a estrategia que sempre ganha eh vc ser o segundo
> jogador e tirar uma pedra de cada vez. 
> 
> 
> 
> --- Johann Peter Gustav Lejeune Dirichlet
> <[EMAIL PROTECTED]> escreveu:
> 
> > Bem, neste tipo de proposicao, quando se fala em
> > estrategia vencedora, ela deve valer para todos os
> > casos, e nao para "os casos de vacilo" do
> > adversario.
> > 
> > 
> > Mas enfim...
> >  Há uma estrategia que vale em todos os casos de
> > pilhas de pedras.
> > Vamos colocar um caso diferente deste:
> > as pilhas tem 1,2,3,4,5,6,7.
> > 
> > Ou, como todo bom computeiro, podemos escrever
> estes
> > valores em binario:
> > 
> > 001
> > 010
> > 011
> > 100
> > 101
> > 110
> > 111
> > 
> > Agora vamos somá-las, de uma maneira nem um pouco
> > convencional:
> > 
> > 001
> > 010
> > 011
> > 100
> > 101
> > 110
> > 111
> > ***+
> >444
> > 
> > Veja que todas as somas deram pares. Com isto, a
> > pessoa que jogar agora perdeu o jogo(isso se você
> e
> > o
> > seu adversario nao vacilarem, como eu estou
> > supondo).
> > 
> > Suponha que você, na sua vez de jogar, ciente
> deste
> > fato fatídico, tira 3 pedras do montinho de 7. 
> > Agora temos esta distribuicao:
> > 
> > 
> > 001
> > 010
> > 011
> > 100
> > 101
> > 110
> > 010
> > ***+
> > 343
> > 
> > Como o 3 e o outro 3 (ensanduichando o 4) sao
> > impares,
> > a ideia sera transforma-los em numeros pares, para
> > assim te manter no desespero, hahaha!
> > Que tal tirar 101? De fato, 
> > 
> > 343
> > 101
> > ***-
> > 242
> > 
> > Agora e so encontrar de onde tirar 101(ou 5,
> > interprte
> > como quiser).
> > Fácil: 
> > 
> > 001
> > 010
> > 011
> > 100
> > 101 -- Esvazie essa!
> > 110
> > 010
> > 
> > Veja que a subtracao tambem nao e convencional :P
> > Aí teremos algo como
> > 
> > 001
> > 010
> > 010
> > 011
> > 100
> > 110
> > ***+
> >242
> > 
> > E assim vai. Com esta estrategia voce estara
> fadado
> > a
> > perdiçao, hahahaha(risadas mais malignas aqui...).
> > 
> > Mas aplicando neste caso (7,7), da o que voce
> disse:
> > sempre tirar para deixar os montes iguais.
> >  
> > 
> > --- Chicao Valadares
> <[EMAIL PROTECTED]>
> > escreveu:
> > 
> > > > Existem duas pilhas com 7 pedras cada. Na sua
> > vez,
> > > > um jogador pode retirar 
> > > > quantas pedras ele quiser, mas somente de uma
> > das
> > > > pilhas. O perdedor é o 
> > > > jogador que não puder jogar. Quem tem a
> > estratégia
> > > > vencedora?
> > > 
> > > - Note que, se em um momento qualquer de uma
> nova
> > > rodada  o jogador X tiver mais pedras que o
> > jogador
> > > Y,
> > > basta o jogador X tirar uma pedra de cada vez e
> > vice
> > > versa.Ou seja , espera-se o vacilo de outro
> > jogador
> > > tirando mais d euma pedra.
> > > 
> > > - Sabendo-se disso entao o jogador X e o jogador
> Y
> > > resolvem tirar uma pedra de cada vez(jogador x
> > > sempre
> > > comeca jogando em uma rodada).Sendo assim ,
> sempre
> > o
> > > jogador Y ganha, pois na vez do jogador X ele
> nao
> > > tera
> > > mais pedras pra jogar.
> > > 
> > > Enfim basta ser o segundo jogador e sempre tirar
> > uma
> > > pedra de cada vez pra sempre ganhar.
> > > 
> > > Sendo o primeiro a jogar, vai depender das
> > > circunstancias do jogo.  









___ 
Novo Yahoo! Messenger com voz: ligações, Yahoo! Avatars, novos emoticons e 
muito mais. Instale agora! 
www.yahoo.com.br/messenger/
=
Instruções para entrar na lista, sair da lista e usar a lista em
http://www.mat.puc-rio.br/~nicolau/olimp/obm-l.html
=


Re: [obm-l] HABILIDADE CRIATIVA!

2005-09-28 Por tôpico Rogerio Ponce
Olá Jorge e pessoal da lista,

separe 10 fichas quaisquer em um grupo e as restantes
(127) no outro grupo. Agora inverta as 10 fichas do 1o
grupo. É o suficiente.

Abraços,
Rogerio Ponce.




--- Jorge Luis Rodrigues e Silva Luis
<[EMAIL PROTECTED]> escreveu:

> 
> Sobre uma mesa há 137 fichas iguais, cada uma com o
> lado vermelho e outro 
> azul, sendo que 10 estão com o lado vermelho para
> cima e as outras com o 
> lado azul. Você está de olhos vendados e deve
> separar as fichas em dois 
> grupos, cada um com a mesma quantidade de fichas
> vermelhas. Você pode virar 
> as fichas se necessário. Como fazer?
> 


__
Faça ligações para outros computadores com o novo Yahoo! Messenger 
http://br.beta.messenger.yahoo.com/ 
=
Instruções para entrar na lista, sair da lista e usar a lista em
http://www.mat.puc-rio.br/~nicolau/olimp/obm-l.html
=


Re: [obm-l] PELO SIM, PELO NÃO!

2005-09-20 Por tôpico Rogerio Ponce
Olá pessoal,
quase não aguento mais esse problema, mas não dá para
ignorar que minhas perguntas podem ser simplificadas,
da seguinte forma:

Pergunte a "A":
- Se daqui a duas perguntas, eu lhe perguntar
 "Existe um honesto entre tais fulanos?"
  você me responderá um SIM?

A resposta obtida sempre será a indicação correta, e a
pesquisa binária pode ser feita sem dificuldade
alguma.

Portanto, com 3 perguntas assim, é possível localizar
o honesto dentre 8 participantes.

[]'s
Rogerio Ponce

PS: Juro que não escrevo mais nada a respeito desta
questão.


-----

--- Rogerio Ponce escreveu:

> Olá Nicolau,
> na verdade, dá para superpor duas vezes (em cada
> pergunta) a política que eu sugeri, de modo a sempre
> obter a verdade.
> 
> Em outras palavras, se com 2 perguntas aninhadas, a
> gente consegue um "inversor", com 4 perguntas
> aninhadas, a gente sempre obtém a verdade.
> 
> E então, mesmo sem a auto-referência, a gente
> consegue
>  distinguir o honesto entre 8 pessoas, fazendo uma
> pesquisa binária desde o início.
> 
> O exemplo é um "pouquinho" enrolado, mas acho que
> funciona:
> 
> Pergunte a "A":
> - Se minha próxima pergunta a você for
>  "Se minha próxima pergunta a você for
> ""Se minha próxima pergunta a você for
>   """Existe um honesto entre tais
> fulanos?"""
> você me responderá um SIM?""
>  você me responderá um SIM?"
>   você me responderá um SIM?
> 
> 
> []s,
> Rogerio Ponce
> 
> 
> 
> --- Rogerio Ponce <[EMAIL PROTECTED]>
> escreveu:
> 
> > Olá Nicolau,
> > esse solução (resolvendo para 8) também é
> > interessante
> > - aliás, é A MAIS INTERESSANTE -, apesar de eu
> > também
> > achar um pouco "apelativa" pela auto-referência.
> > 
> > 
> > O que imaginei anteriormente, resolveria apenas
> para
> > 5
> > participantes (A,B,C,D,E), da seguinte forma:
> > 
> > Pergunte a "A":
> >  - Se minha próxima pergunta a você for "Existe
> > apenas
> > 1 honesto entre vocês?" , você me responderá um
> > "SIM"?
> > 
> > O honesto responderá SIM, e um desonesto
> responderá
> > NÃO. Supondo que "A" seja desonesto, agora você
> faz
> > a
> > seguinte pergunta a "A":
> > 
> > - Se minha próxima pergunta a você for "O honesto
> se
> > encontra entre B e C?" , você me responderá um
> > "SIM"?
> > 
> > Se a resposta for "NÃO" , então o honesto é B ou
> C.
> > Caso contrário, o honesto é D ou E.
> > Supondo que tenha respondido "NÃO", agora você
> > pergunta a "A":
> > 
> > - Se minha próxima pergunta a você for "O honesto
> é
> > B?", você me responderá um "SIM"?
> > 
> > Se a resposta for "NÃO" , o honesto é B, caso
> > contrário é C.
> > 
> > As outras derivações se resolvem do mesma modo,
> > sempre
> > usando a "dupla filtragem pelo desonesto", de
> forma
> > a
> > sempre obter a resposta invertida.
> > 
> > 
> > Mas como falei, essa minha solução ficou "na
> > poeira",
> > pois só consegue resolver para 5 pessoas...
> > 
> > []s,
> > Rogerio Ponce.
> > 
> > 
> > --- "Nicolau C. Saldanha" <[EMAIL PROTECTED]>
> > escreveu:
> > 
> > > On Wed, Sep 14, 2005 at 09:59:01PM -0300,
> Rogerio
> > > Ponce wrote:
> > > > Olá Nicolau,
> > > > sua solução é bonita porque resolve para
> > qualquer
> > > número de pessoas.
> > > > Mas, e se todos (como sugeriu o Chicão) só
> > puderem
> > > responder "sim" ou "não" a
> > > > qualquer questão?
> > > >  
> > > > Parece-me que - neste caso de apenas 5
> > > participantes - ainda é possível
> > > > resolver com apenas 3 perguntas.
> > > 
> > > Acho que dá até com 8 participantes, mas só com
> um
> > > pouco de apelação.
> > > Digamos que os participantes se chamam
> > > 000, 001, 010, 011, 100, 101, 110, 111.
> > > As perguntas seriam:
> > > 
> > > "Considere a seguinte afirmação:
> > > 'A sua resposta para esta pergunta será
> verdadeira
> > > se e somente se
> > > o primeiro algarismo do nome do honesto é 1.';
> > > a afirmação é verdadeira?"
> > > 
> 

Re: [obm-l] A VERDADE!

2005-09-20 Por tôpico Rogerio Ponce
Olá Jorge Luis,
como de hábito, vc enviou mais um problema
interessante.

Achei natural que o mais atraente fosse considerarmos
que qualquer resposta devia ser apenas "SIM" ou "NAO".
Entretanto, se fosse dada a liberdade de qualquer tipo
de resposta, então acho que resolveríamos a questão
(para qualquer número de desonestos) com apenas uma
pergunta:

 "Quem tu rejeitarias se, daqui a duas perguntas, eu
pedisse tua concordância sobre ele ser um desonesto?"

Abraços,
Rogerio Ponce







--- Jorge Luis Rodrigues e Silva Luis
<[EMAIL PROTECTED]> escreveu:
> ... A minha contribuição se limitará a
> enviar "na íntegra" a 
> resolução de um professor da FGV, que aliás é
> contemporâneo do Elon.
> 
> É preciso fazer duas perguntas, a qualquer um deles.
> A primeira é: "Você é o 
> honesto?" Se a resposta for "sim", pergunte então:
> "Quem é o honesto?" Se 
> for ele, apontará a si mesmo. Se não for, apontará o
> verdadeiro, pois estava 
> mentindo na primeira resposta. Se a primeira
> resposta for "não", ele é um 
> mentiroso que está dizendo a verdade. A próxima
> resposta será mentira. 
> Pergunte então: "Quem não é o honesto?" E ele
> apontará o honesto.
> 







___ 
Novo Yahoo! Messenger com voz: ligações, Yahoo! Avatars, novos emoticons e 
muito mais. Instale agora! 
www.yahoo.com.br/messenger/
=
Instruções para entrar na lista, sair da lista e usar a lista em
http://www.mat.puc-rio.br/~nicolau/olimp/obm-l.html
=


Re: [obm-l] PELO SIM, PELO NÃO!

2005-09-18 Por tôpico Rogerio Ponce
Olá Nicolau,
a primeira resposta de um desonesto pode ser o que ele preferir (verdadeira ou mentirosa), e a partir daí, ele sempre inverte, conforme o enunciado esclareceu perfeitamente.
 
Repare que se na pergunta atual ele for mentiroso, então na próxima ele seria verdadeiro. Como na próxima ele me responderia "SIM" (pois estaria sendo verdadeiro) , então ele me diz (na pergunta atual) um "NÃO", pois no momento ele é mentiroso.
 
Se no entanto, ele no momento for verdadeiro, então , na próxima pergunta ele seria mentiroso , e diria "NÃO" .  E é isso que ele me conta na pergunta atual, pois estará sendo verdadeiro.
 
Portanto, o mentiroso sempre responderá NÃO àquela pergunta longa, enquanto o honesto sempre responderá SIM.
 
[]s
Rogerio Ponce.
 
"Nicolau C. Saldanha" <[EMAIL PROTECTED]> escreveu:
On Fri, Sep 16, 2005 at 05:35:18PM +, Rogerio Ponce wrote:> Olá Nicolau,> esse solução (resolvendo para 8) também é interessante> - aliás, é A MAIS INTERESSANTE -, apesar de eu também> achar um pouco "apelativa" pela auto-referência.> > > O que imaginei anteriormente, resolveria apenas para 5> participantes (A,B,C,D,E), da seguinte forma:> > Pergunte a "A":> - Se minha próxima pergunta a você for "Existe apenas> 1 honesto entre vocês?" , você me responderá um "SIM"?> > O honesto responderá SIM, e um desonesto responderá> NÃO.Acho que voce está supondo que a primeira resposta de um desonestoé sempre falsa. Eu não interpretei o enunciado desta forma (nem, acho,a maioria dos outros que comentaram a questão). Se for assim a suasolução é correta e o prob!
lema todo
 fica bem mais fácil.[]s, N.__Converse com seus amigos em tempo real com o Yahoo! Messenger http://br.download.yahoo.com/messenger/ 

Re: [obm-l] PELO SIM, PELO NÃO!

2005-09-16 Por tôpico Rogerio Ponce
Olá Nicolau,
na verdade, dá para superpor duas vezes (em cada
pergunta) a política que eu sugeri, de modo a sempre
obter a verdade.

Em outras palavras, se com 2 perguntas aninhadas, a
gente consegue um "inversor", com 4 perguntas
aninhadas, a gente sempre obtém a verdade.

E então, mesmo sem a auto-referência, a gente consegue
 distinguir o honesto entre 8 pessoas, fazendo uma
pesquisa binária desde o início.

O exemplo é um "pouquinho" enrolado, mas acho que
funciona:

Pergunte a "A":
- Se minha próxima pergunta a você for
 "Se minha próxima pergunta a você for
""Se minha próxima pergunta a você for
  """Existe um honesto entre tais fulanos?"""
você me responderá um SIM?""
 você me responderá um SIM?"
  você me responderá um SIM?


[]s,
Rogerio Ponce



--- Rogerio Ponce <[EMAIL PROTECTED]>
escreveu:

> Olá Nicolau,
> esse solução (resolvendo para 8) também é
> interessante
> - aliás, é A MAIS INTERESSANTE -, apesar de eu
> também
> achar um pouco "apelativa" pela auto-referência.
> 
> 
> O que imaginei anteriormente, resolveria apenas para
> 5
> participantes (A,B,C,D,E), da seguinte forma:
> 
> Pergunte a "A":
>  - Se minha próxima pergunta a você for "Existe
> apenas
> 1 honesto entre vocês?" , você me responderá um
> "SIM"?
> 
> O honesto responderá SIM, e um desonesto responderá
> NÃO. Supondo que "A" seja desonesto, agora você faz
> a
> seguinte pergunta a "A":
> 
> - Se minha próxima pergunta a você for "O honesto se
> encontra entre B e C?" , você me responderá um
> "SIM"?
> 
> Se a resposta for "NÃO" , então o honesto é B ou C.
> Caso contrário, o honesto é D ou E.
> Supondo que tenha respondido "NÃO", agora você
> pergunta a "A":
> 
> - Se minha próxima pergunta a você for "O honesto é
> B?", você me responderá um "SIM"?
> 
> Se a resposta for "NÃO" , o honesto é B, caso
> contrário é C.
> 
> As outras derivações se resolvem do mesma modo,
> sempre
> usando a "dupla filtragem pelo desonesto", de forma
> a
> sempre obter a resposta invertida.
> 
> 
> Mas como falei, essa minha solução ficou "na
> poeira",
> pois só consegue resolver para 5 pessoas...
> 
> []s,
> Rogerio Ponce.
> 
> 
> --- "Nicolau C. Saldanha" <[EMAIL PROTECTED]>
> escreveu:
> 
> > On Wed, Sep 14, 2005 at 09:59:01PM -0300, Rogerio
> > Ponce wrote:
> > > Olá Nicolau,
> > > sua solução é bonita porque resolve para
> qualquer
> > número de pessoas.
> > > Mas, e se todos (como sugeriu o Chicão) só
> puderem
> > responder "sim" ou "não" a
> > > qualquer questão?
> > >  
> > > Parece-me que - neste caso de apenas 5
> > participantes - ainda é possível
> > > resolver com apenas 3 perguntas.
> > 
> > Acho que dá até com 8 participantes, mas só com um
> > pouco de apelação.
> > Digamos que os participantes se chamam
> > 000, 001, 010, 011, 100, 101, 110, 111.
> > As perguntas seriam:
> > 
> > "Considere a seguinte afirmação:
> > 'A sua resposta para esta pergunta será verdadeira
> > se e somente se
> > o primeiro algarismo do nome do honesto é 1.';
> > a afirmação é verdadeira?"
> > 
> > É fácil verificar que se a resposta for SIM (resp.
> > NÃO)
> > então o primeiro algarismo do nome do honesto é 1
> > (resp. 0),
> > independentemente da resposta ser verdadeira ou
> > falsa.
> > Isto é parecido com o truque apresentado pelo Gugu
> > mas um pouco diferente
> > (e eu acho que agora correto). Note que a pergunta
> é
> > duplamente
> > problemática: é auto-referente e pergunta sobre o
> > futuro.
> > É muito fácil com este tipo de 'golpe baixo'
> > produzir perguntas
> > irrespondíveis, como
> > "Considere a seguinte afirmação:
> > 'A sua resposta para esta pergunta será verdadeira
> > se e somente se
> > a sua resposta será NÃO.';
> > e afirmação é verdadeira?"
> > 
> > Naturalmente, a segunda e terceira pergunta são,
> > respectivamente:
> > 
> > "Considere a seguinte afirmação:
> > 'A sua resposta para esta pergunta será verdadeira
> > se e somente se
> > o segundo algarismo do nome do honesto é 1.';
> > e afirmação é verdadeira?"
> > 
> > "Considere a seguinte afirmação:
> > 'A sua resposta para esta pergunta

Re: [obm-l] PELO SIM, PELO NÃO!

2005-09-16 Por tôpico Rogerio Ponce
Olá Nicolau,
esse solução (resolvendo para 8) também é interessante
- aliás, é A MAIS INTERESSANTE -, apesar de eu também
achar um pouco "apelativa" pela auto-referência.


O que imaginei anteriormente, resolveria apenas para 5
participantes (A,B,C,D,E), da seguinte forma:

Pergunte a "A":
 - Se minha próxima pergunta a você for "Existe apenas
1 honesto entre vocês?" , você me responderá um "SIM"?

O honesto responderá SIM, e um desonesto responderá
NÃO. Supondo que "A" seja desonesto, agora você faz a
seguinte pergunta a "A":

- Se minha próxima pergunta a você for "O honesto se
encontra entre B e C?" , você me responderá um "SIM"?

Se a resposta for "NÃO" , então o honesto é B ou C.
Caso contrário, o honesto é D ou E.
Supondo que tenha respondido "NÃO", agora você
pergunta a "A":

- Se minha próxima pergunta a você for "O honesto é
B?", você me responderá um "SIM"?

Se a resposta for "NÃO" , o honesto é B, caso
contrário é C.

As outras derivações se resolvem do mesma modo, sempre
usando a "dupla filtragem pelo desonesto", de forma a
sempre obter a resposta invertida.


Mas como falei, essa minha solução ficou "na poeira",
pois só consegue resolver para 5 pessoas...

[]s,
Rogerio Ponce.


--- "Nicolau C. Saldanha" <[EMAIL PROTECTED]>
escreveu:

> On Wed, Sep 14, 2005 at 09:59:01PM -0300, Rogerio
> Ponce wrote:
> > Olá Nicolau,
> > sua solução é bonita porque resolve para qualquer
> número de pessoas.
> > Mas, e se todos (como sugeriu o Chicão) só puderem
> responder "sim" ou "não" a
> > qualquer questão?
> >  
> > Parece-me que - neste caso de apenas 5
> participantes - ainda é possível
> > resolver com apenas 3 perguntas.
> 
> Acho que dá até com 8 participantes, mas só com um
> pouco de apelação.
> Digamos que os participantes se chamam
> 000, 001, 010, 011, 100, 101, 110, 111.
> As perguntas seriam:
> 
> "Considere a seguinte afirmação:
> 'A sua resposta para esta pergunta será verdadeira
> se e somente se
> o primeiro algarismo do nome do honesto é 1.';
> a afirmação é verdadeira?"
> 
> É fácil verificar que se a resposta for SIM (resp.
> NÃO)
> então o primeiro algarismo do nome do honesto é 1
> (resp. 0),
> independentemente da resposta ser verdadeira ou
> falsa.
> Isto é parecido com o truque apresentado pelo Gugu
> mas um pouco diferente
> (e eu acho que agora correto). Note que a pergunta é
> duplamente
> problemática: é auto-referente e pergunta sobre o
> futuro.
> É muito fácil com este tipo de 'golpe baixo'
> produzir perguntas
> irrespondíveis, como
> "Considere a seguinte afirmação:
> 'A sua resposta para esta pergunta será verdadeira
> se e somente se
> a sua resposta será NÃO.';
> e afirmação é verdadeira?"
> 
> Naturalmente, a segunda e terceira pergunta são,
> respectivamente:
> 
> "Considere a seguinte afirmação:
> 'A sua resposta para esta pergunta será verdadeira
> se e somente se
> o segundo algarismo do nome do honesto é 1.';
> e afirmação é verdadeira?"
> 
> "Considere a seguinte afirmação:
> 'A sua resposta para esta pergunta será verdadeira
> se e somente se
> o terceiro algarismo do nome do honesto é 1.';
> a afirmação é verdadeira?"
> 
> Note que com estas perguntas podem ser dirigidas a
> qualquer um.
> Você determina quem é o honesto mas, paradoxalmente,
> fica eternamente
> sem saber se as respostas que você ouviu eram
> verdadeiras ou falsas.
> 
> Acredito que sem este tipo de apelação é impossível
> resolver o problema
> original, com 5 pessoas chamadas A, B, C, D, E.
> 
> De fato, três perguntas com resposta SIM ou NÃO
> criam 8 possíveis
> resultados (isto é verdade mesmo se as perguntas
> dependerem das
> respostas anteriores). Ora, sem algum tipo de
> apelação você esperaria
> que ao resolver o problema descobrisse não apenas
> quem é o honesto,
> mas se as pessoas com quem você falou estavam
> mentindo ou não.
> Mesmo se você dirigir todas as perguntas à mesma
> pessoa (digamos, A)
> isto criaria 9 casos:
> 
> A é honesto.
> B é honesto e A respondeu VFV.
> B é honesto e A respondeu FVF.
> C é honesto e A respondeu VFV.
> C é honesto e A respondeu FVF.
> D é honesto e A respondeu VFV.
> D é honesto e A respondeu FVF.
> E é honesto e A respondeu VFV.
> E é honesto e A respondeu FVF.
> 
> Ora, com 8 possíveis resultados é impossível decidir
> entre 9 casos.
> 
> []s, N.
> 

__
Converse com seus amigos em tempo real com o Yahoo! Messenger 
http://br.download.yahoo.com/messenger/ 
=
Instruções para entrar na lista, sair da lista e usar a lista em
http://www.mat.puc-rio.br/~nicolau/olimp/obm-l.html
=


Re: [obm-l] PELO SIM, PELO NÃO!

2005-09-14 Por tôpico Rogerio Ponce
Olá Nicolau,
sua solução é bonita porque resolve para qualquer número de pessoas.
Mas, e se todos (como sugeriu o Chicão) só puderem responder "sim" ou "não" a qualquer questão?
 
Parece-me que - neste caso de apenas 5 participantes - ainda é possível resolver com apenas 3 perguntas.
 
[]'s
Rogerio Ponce
 
"Nicolau C. Saldanha" <[EMAIL PROTECTED]> escreveu:
On Wed, Sep 14, 2005 at 05:06:24PM -0300, Nicolau C. Saldanha wrote:> On Wed, Sep 14, 2005 at 12:29:42PM -0400, Qwert Smith wrote:> > Sao necessarias pelo menos 2 perguntas.> > > > Escolha um dos individuos e peca a ele que identifique os desonestos.> > Logo em seguida faca o mesmo pedido ao mesmo individuo.> > > > Se ele for o honsto suas respostas seram iguais e ele nao se acusa nunca.> > Caso contrario suas respostas serao diferentes. Basta entao vc se valer da > > resposta em que ele se acusa como um dos desonestos, ja que nessa resposta > > ele esta falando a verdade.> > Acho que nao. Se as perguntas forem dirigidas a A e o honesto for B,> ele pode dar as duas seguintes respostas (em qualquer ordem):> > "O honesto e o B." (verdade)> "!
O honesto
 e o C." (mentira)> > Voce so pode concluir que o honesto e ou B ou C.Só para dar um tom mais positivo às minhas contribuições:você sempre pode resolver o problema com as 3 seguintes perguntas:"Quem é o honesto?", "Quem é o honesto?", "Você é honesto?".Se as duas primeiras perguntas receberem a mesma resposta,nosso interlocutor é honesto e a terceira pergunta é supérflua.Se as duas primeiras perguntas receberem respostas diferentesentão sabemos que nosso interlocutor não é honesto e portantocom a terceira resposta ele indicará qual das duas primeirasrespostas era a correta.[]s, N.__Converse com seus amigos em tempo real com o Yahoo! Messenger http://br.download.yahoo.com/messenger/ 

Re: [obm-l] Problema do lixeiro - adendo

2005-09-14 Por tôpico Rogerio Ponce
Olá Demétrius,
obrigado pelos links, mas não é necessário nenhum
conhecimento de grafos para se resolver esse problema.

A forma de abordá-lo é que é o "x da questão" : nada
de força bruta, seguindo o impulso de se tentar
seguidamente um caminho melhor que o outro.

Depois que vc pensar o suficiente a respeito, apenas
olhando para a figura, vai gastar menos de 1 minuto
com o lápis para desenhar um caminho ótimo (que
diga-se de passagem, mede 35x100m).

Abraços,
Rogerio Ponce.




--- Demétrius <[EMAIL PROTECTED]> escreveu:

> É o mesmo problema do carteiro chinês, não?!?!
> 
> Tem muito tempo que não executo estes algoritmos e
> portanto me sinto um pouco desconfortável em entrar
> em
> detalhes...
> 
> Utilize um algoritmos de busca.
> 
> No google vc acha vários tipos de soluções!
> Aqui algumas que ele me retornou... 
> 
> http://arxiv.org/pdf/cs.MS/0505031
> http://pt.wikipedia.org/wiki/Teoria_dos_grafos
>
http://www.universiabrasil.net/mit/6/6281J/pdf/f01-lec14.pdf
> 
> Abraços,
> Demétrius P. de Miranda
> 
> 
> --- Rogerio Ponce <[EMAIL PROTECTED]>
> escreveu:
> 
> > As ruas externas tambem fazem parte, ou seja, a
> > distância a ser varrida e' de 31x100m.
> > 
> >  
> > --- Rogerio Ponce <[EMAIL PROTECTED]>
> > escreveu: 
> > > Ola' pessoal,
> > > um lixeiro precisa varrer todas as ruas dos 12
> > > quarteirões abaixo, comecando numa esquina
> > qualquer,
> > > e
> > > tambem terminando em alguma esquina.
> > > Considerando-se que o lado do quarteirão mede
> > 100m,
> > > qual a distancia minima que ele deve percorrer
> > para
> > > varrer todas as ruas?
> > > 
> > > Abracos,
> > > Rogerio Ponce.
> > > 
> > > a---b---c---d---e
> > > |   |   |   |   |
> > > |   |   |   |   |
> > > f---g---h---i---j
> > > |   |   |   |   |
> > > |   |   |   |   |
> > > k---l---m---n---o
> > > |   |   |   |   |
> > > |   |   |   |   |
> > > p---q---r---s---t
> > > 









___ 
Yahoo! Messenger com voz: PROMOÇÃO VOCÊ PODE LEVAR UMA VIAGEM NA CONVERSA. 
Participe! www.yahoo.com.br/messenger/promocao
=
Instruções para entrar na lista, sair da lista e usar a lista em
http://www.mat.puc-rio.br/~nicolau/olimp/obm-l.html
=


Re: [obm-l] Problema do lixeiro - adendo

2005-09-13 Por tôpico Rogerio Ponce
As ruas externas tambem fazem parte, ou seja, a
distância a ser varrida e' de 31x100m.

 
--- Rogerio Ponce <[EMAIL PROTECTED]>
escreveu: 
> Ola' pessoal,
> um lixeiro precisa varrer todas as ruas dos 12
> quarteirões abaixo, comecando numa esquina qualquer,
> e
> tambem terminando em alguma esquina.
> Considerando-se que o lado do quarteirão mede 100m,
> qual a distancia minima que ele deve percorrer para
> varrer todas as ruas?
> 
> Abracos,
> Rogerio Ponce.
> 
> a---b---c---d---e
> |   |   |   |   |
> |   |   |   |   |
> f---g---h---i---j
> |   |   |   |   |
> |   |   |   |   |
> k---l---m---n---o
> |   |   |   |   |
> |   |   |   |   |
> p---q---r---s---t
> 
> 
> 
> 
>   
> 
> 
>   
>   
>
___
> 
> Yahoo! Messenger com voz: PROMOÇÃO VOCÊ PODE LEVAR
> UMA VIAGEM NA CONVERSA. Participe!
> www.yahoo.com.br/messenger/promocao
>
=
> Instruções para entrar na lista, sair da lista e
> usar a lista em
> http://www.mat.puc-rio.br/~nicolau/olimp/obm-l.html
>
=
>  

__
Converse com seus amigos em tempo real com o Yahoo! Messenger 
http://br.download.yahoo.com/messenger/ 
=
Instruções para entrar na lista, sair da lista e usar a lista em
http://www.mat.puc-rio.br/~nicolau/olimp/obm-l.html
=


[obm-l] Problema do lixeiro

2005-09-13 Por tôpico Rogerio Ponce
Ola' pessoal,
um lixeiro precisa varrer todas as ruas dos 12
quarteirões abaixo, comecando numa esquina qualquer, e
tambem terminando em alguma esquina.
Considerando-se que o lado do quarteirão mede 100m,
qual a distancia minima que ele deve percorrer para
varrer todas as ruas?

Abracos,
Rogerio Ponce.

a---b---c---d---e
|   |   |   |   |
|   |   |   |   |
f---g---h---i---j
|   |   |   |   |
|   |   |   |   |
k---l---m---n---o
|   |   |   |   |
|   |   |   |   |
p---q---r---s---t









___ 
Yahoo! Messenger com voz: PROMOÇÃO VOCÊ PODE LEVAR UMA VIAGEM NA CONVERSA. 
Participe! www.yahoo.com.br/messenger/promocao
=
Instruções para entrar na lista, sair da lista e usar a lista em
http://www.mat.puc-rio.br/~nicolau/olimp/obm-l.html
=


Re: [obm-l] Gabarito no site

2005-09-06 Por tôpico Gabriel Ponce
Oi Nelly , as provas ainda não estão no site!
Obrigado
Gabriel

Em 06/09/05, Olimpiada Brasileira de Matematica<[EMAIL PROTECTED]> escreveu:
> Caros amigos da lista,
> 
> As provas e gabaritos já estão disponíveis
> no site da OBM.
> Confiram!.
> www.obm.org.br
> 
> Abraços, Nelly
> =
> Instruções para entrar na lista, sair da lista e usar a lista em
> http://www.mat.puc-rio.br/~nicolau/olimp/obm-l.html
> =
>

=
Instruções para entrar na lista, sair da lista e usar a lista em
http://www.mat.puc-rio.br/~nicolau/olimp/obm-l.html
=


Re: [obm-l] Problema das velas

2005-09-03 Por tôpico Rogerio Ponce
Olá pessoal,
se eu fiquei sem luz durante o tempo T , então uma vela queimou T/3 , e a outra T/5 de seu comprimento inicial.
 
Logo, 2*(1-T/3) = (1-T/5) , o que leva a  T = 15/7 horas .
 
[]'s
Rogerio Ponce
saulo nilson <[EMAIL PROTECTED]> escreveu:

5/3=(l-x)/(l-2x)
l=13x/2
 
5l-10x=3l-3x
2l=7x
x=2/7l 
On 9/2/05, Osvaldo Mello Sponquiado <[EMAIL PROTECTED]> wrote: 

Velocidade de queima da primeira vela: v1
Velocidade de queima da segunda vela: v2
Comprimento da vela: l
Tempo que faltou luz: t
v1=l/3
v2=l/5
Sendo assim v1>v2, logo a vela 1 queimou mais do que a vela 2, dai quando voltou a luz a vela 1 estava com x de comprimento e a vela dois com 2x, logo
 
 
l/3=(l-x)/t   (*)
l/5=(l-2x)/t
 
Dividindo, membro a membro, as igualdades vem:
 
5/3=(l-x)/(l-2x)
l=13x/2
 
Substituindo este resultado em *, vem:
 
13x/6=11x/2t=>t=33/13 horas que dá um poquinho mais do que duas horas e meia.
 
 
[]'s vardim.
 
> Faltou luz. Daí acendi 2 velas. Quando a luz voltou, apaguei as velas. 
> Sendo elas do mesmo tamanho, a primeira tinha autonomia de 3 horas, 
> enquanto que a outra tinha autonomia de 5 horas. Depois de apagadas 
> notei que o resto de uma tinha o dobro do resto de outra. Quanto tempo 
> eu fiquei sem luz? 
> 
> 

Atenciosamente, 

Osvaldo Mello Sponquiado 
Engenharia Elétrica, 2ºano 
UNESP - Ilha Solteira 
__Converse com seus amigos em tempo real com o Yahoo! Messenger http://br.download.yahoo.com/messenger/ 

Re: [obm-l] O Problema do jipe

2005-09-03 Por tôpico Rogerio Ponce
Olá Bernardo e Alamir,
procurem pelo "problema do camelo", já discutido aqui na lista.
[]'s
Rogerio PonceBernardo Freitas Paulo da Costa <[EMAIL PROTECTED]> escreveu:
Eu acho que está meio difícil... Se você tiver espaço no jipe ra levar9 vezes o tanque de gasolina na traseira, está resolvido (bom, leve"um pouco mais", que n~ao faz mal)Se você tiver mais de um jipe, daí você tem que ver quanta gasolinavocê precisa ter "em cada ponto do trajeto" pra ter um jipe que chegueao fim do mesmo. Isso quer dizer que você vai usar jipes que n~aoconsomem todo o tanque, mas que param antes disso e transferem o quesobrou para um outro, que continua. Eu acho que é isso. Uma idéia:Para ter um que chege ao final, você precisa de um jipe "completo" a9x320 km, certo? Ent~ao, você tem que ter (pelo menos) mais um jipenesse local, com zero gasolina. Suponha (n~ao sei se é o melhor), quefoi passado meio tanque de combustível de uma pra outra. Assim, temosdois jipes a 9x320, com meio tanque. Ou seja, você t!
eria que
 ter doisjipes completos a 8.5x320 km. E por aí vai (bom, depois vocêgeneralisa, pra fazer as contas)Fui,-- Bernardo Freitas Paulo da CostaOn 9/3/05, Alamir Rodrigues <[EMAIL PROTECTED]>wrote:> > Se trata de um famoso enigma logístico da Segunda Guerra Mundial. Em> essência, esse problema pede que você cruze o deserto do Saara, com 3200> quilômetros de extensão, mas o tanque de gasolina do veículo só tem> capacidade para viajar 320 quilometros. Como atravessar o deserto então?=Instruções para entrar na lista, sair da lista e usar a lista emhttp://www.mat.puc-rio.br/~nicolau/olimp/obm-l.html=
		 
Yahoo! Messenger com voz: PROMOÇÃO VOCÊ PODE LEVAR UMA VIAGEM NA CONVERSA. Participe!

[obm-l] Re: [obm-l] divisor

2005-06-23 Por tôpico gabriel . ponce

oi .( aqui eu ultilizei a.b como sendo a vezes b)
 Sem querer você confundiu-se dizendo que 2^16 . 2 == 2.2 (mod3).
Isto não é verdade, aliás como já havia colocado o professor Nnicolau, 2^par
== 1(mod 3)pois 2== -1(mod 3).Logo 
2^16.2 == 1 .2 (mod 3). obs: pode colocar 2^16.2== -1(mod 3) se você preferir.
Deste modo N == 2 - 2 - 2 - 1 == 0 (mod 3).
Nota: Apenas para constar> 
2^17( 2^16 - 2^2 - 1) - 1 = 131072(65536 -4 - 1)-1  = 8589279231 = 1983
. 4331457 
-- Mensagem original --

>
>
>"Nicolau C. Saldanha" <[EMAIL PROTECTED]> escreveu:
>On Tue, Jun 21, 2005 at 03:50:14PM -0300, claudio.buffara wrote:
>> > On Mon, Jun 20, 2005 at 10:55:04PM -0300, fgb1 wrote:
>> > > Pessoal, preciso de ajuda nessa:
>> > >
>> > > Um fator de 2^33 - 2^19 - 2^17 -1, entre 1000 e 5000 é:
>> > > a) 1993
>> > > b) 1992
>> > > c) 1983
>> > > d) 1982
>> > > e) 1972

>> N = 2^33 - 2^19 - 2^17 - 1 eh obviamente impar, o que elimina as alternativas
>> b, d, e. Olhando mod 3, e levando em conta que 2^par == 1 e 2^impar ==
>2,
>> teremos que N == 2 - 2 - 2 - 1 == 0 ==> N soh pode ser 1983, pois 1993
>nao eh
>> divisivel por 3.
>
>Não entendi esta solução. E daí que 1993 não seja múltiplo de 3?
>13, 661 e 13*661 = 8593 são divisores de 2^33 - 2^19 - 2^17 - 1 
>mas nenhum deles é múltiplo de 3.
>
>[]s, N.
>
> N = 2^33 - 2^19 - 2^17 - 1 eh obviamente impar, o que elimina as 
>alternativas
>> b, d, e.  Olhando mod 3, e levando em conta que 2^par == 1 e 2^impar

>== 2,
>
>2^17=2^16.2^1==2.2
>> teremos que N == 2 - 2 - 2.2 - 1 == 1 ==> N soh pode ser 1993, pois 
>1983 nao eh.Sendo que
>13, 661 e 13*661 = 8593 são divisores de 2^33 - 2^19 - 2^17 - 1 
>e deixam resto 1 na divisão por 3.
>
> 
>
>=
>Instruções para entrar na lista, sair da lista e usar a lista em
>http://www.mat.puc-rio.br/~nicolau/olimp/obm-l.html
>=
>
>
>
>   
>-
>Yahoo! Acesso Grátis: Internet rápida e grátis. Instale o discador agora!




--
Use o melhor sistema de busca da Internet
Radar UOL - http://www.radaruol.com.br




=
Instruções para entrar na lista, sair da lista e usar a lista em
http://www.mat.puc-rio.br/~nicolau/olimp/obm-l.html
=


[obm-l] Re: [obm-l] irracionalidade do pi

2005-06-18 Por tôpico gabriel . ponce
De fato eu também acho estranho definir o pi como a razão entre o comprmento
e o diametro da circunferência sendo ( o pi irracional )e gostaria de entender
melhor isso!

-- Mensagem original --

>Apesar de ser um assunto, nem tanto, elementar, nossos alunos sempre nos
>fazem perguntas sobre irracionais. Tipo:
> 
>Alguém conhece algum método elementar de demonstrar a irracionalidade do
>número pi (para o ensino médio)?
>
>Se pi é irracional, não traz um certo desconforto definí-lo como a razão
>entre o comprimento e o diâmetro da circunferência? Afinal, quem é irracinal,
>pi ou 2.pi.r.?(pergunta inocente).
>
>É possível dar uma aproximação razoável para a raiz quadrada de pi? como?
>
>Obrigado.
>
>Em tempo, alguém conhece algum sitio onde encontro exercícios e problemas
>com números primos para o ensino fundamental?




--
Use o melhor sistema de busca da Internet
Radar UOL - http://www.radaruol.com.br




=
Instruções para entrar na lista, sair da lista e usar a lista em
http://www.mat.puc-rio.br/~nicolau/olimp/obm-l.html
=


[obm-l] Re: [obm-l] Re: [obm-l] questao 14 do nivel 3 da obm (axu q tem erro no gabarito)

2005-06-14 Por tôpico gabriel . ponce

HEHEHEHE desculpa , faltou o não , a resposta é 5 mesmo viu , hehehe 
desculpa.!!!
-- Mensagem original --

>então , eu tb achei estranho o gabarito , quando eu fiz a prova deu que
>o número que deveria ocupar a posição do 6 (se eu bem me lembro , pois
não
>estou com  a minha prova) era o 5.!!E de fato , 7 é número triângular.
>
>-- Mensagem original --
>
>>Por favor, alguem poderia me confirmar, ou apontar o meu erro, se o gabarito
>>
>>da questao 14 do nivel 3 da obm está errada no site da obm???
>>No gabarito está dizendo q 6 só faz números triangulares com 9 e 4 (com
>os
>>
>>números do relógio), porém, no relógio da questao, aparece o 1 vizinho
>ao
>>6 
>>( 7 nao é número triangular)
>>desde já agradeço
>>
>>Diogo
>>
>>_
>>MSN Messenger: converse online com seus amigos .  
>>http://messenger.msn.com.br
>>
>>=
>>Instruções para entrar na lista, sair da lista e usar a lista em
>>http://www.mat.puc-rio.br/~nicolau/olimp/obm-l.html
>>=
>>
>
>
>
>--
>Use o melhor sistema de busca da Internet
>Radar UOL - http://www.radaruol.com.br
>
>
>
>
>=
>Instruções para entrar na lista, sair da lista e usar a lista em
>http://www.mat.puc-rio.br/~nicolau/olimp/obm-l.html
>=
>



--
Use o melhor sistema de busca da Internet
Radar UOL - http://www.radaruol.com.br




=
Instruções para entrar na lista, sair da lista e usar a lista em
http://www.mat.puc-rio.br/~nicolau/olimp/obm-l.html
=


[obm-l] Re: [obm-l] questao 14 do nivel 3 da obm (axu q tem erro no gabarito)

2005-06-13 Por tôpico gabriel . ponce
então , eu tb achei estranho o gabarito , quando eu fiz a prova deu que
o número que deveria ocupar a posição do 6 (se eu bem me lembro , pois não
estou com  a minha prova) era o 5.!!E de fato , 7 é número triângular.

-- Mensagem original --

>Por favor, alguem poderia me confirmar, ou apontar o meu erro, se o gabarito
>
>da questao 14 do nivel 3 da obm está errada no site da obm???
>No gabarito está dizendo q 6 só faz números triangulares com 9 e 4 (com
os
>
>números do relógio), porém, no relógio da questao, aparece o 1 vizinho
ao
>6 
>( 7 nao é número triangular)
>desde já agradeço
>
>Diogo
>
>_
>MSN Messenger: converse online com seus amigos .  
>http://messenger.msn.com.br
>
>=
>Instruções para entrar na lista, sair da lista e usar a lista em
>http://www.mat.puc-rio.br/~nicolau/olimp/obm-l.html
>=
>



--
Use o melhor sistema de busca da Internet
Radar UOL - http://www.radaruol.com.br




=
Instruções para entrar na lista, sair da lista e usar a lista em
http://www.mat.puc-rio.br/~nicolau/olimp/obm-l.html
=


[obm-l] Re:

2005-06-02 Por tôpico gabriel . ponce

o que significa aqueles pontos de interogação entre as funções e entre o
x e o y?
-- Mensagem original --

>Estou com dúvida em relação à solução dessa questão:
> 
>
>Determine todas as funções f: Â ® Â tais que  f(x + y) ? f(x ? y) = f(x).f(y)
> para x, y Î Â.
>
>Resolução:
>
>Fazendo x=y=0 ® f(0)-f(0)=f(0).f(0) ® f(0)=0
>
>Sabemos que   f(x + y) ? f(x ? y) = f(x).f(y) (1)
>
>Fazendo x=y e y=x ® f(y+x)-f(y-x)=f(x).f(y) (2)
>
>(2)-(1) ® f(y-x)-f(x-y)=0 ® f(y-x)=f(x-y) ® f(y-x)=f((-1)(y-x)), portanto
>a função é par!
>
>Fazendo y=x ® f(2x)-f(0)=f(x).f(x) ® f(x+x)=f(x)^2® f(2x)=f(x)^2
>Fazendo y=2x  ® f(x+2x)-f(x-2x)=f(x).f(2x) ® 
>f(3x)-f(-x)=f(x).f(x)^2®f(3x)-f(x)=f(x)^3®f(3x)=f(x)^3+f(x)
>
>Fazendo x= 2x? e y=x? em (1) : f(2x+x)-f(2x-x)=f(2x).f(x) ® f(3x)-f(x)=
f(x)^2.f(x)
>® f(x)^3+f(x)= f(x)3 ®f(x)=0 para todo x real ?
>
>
>   
>-
>Yahoo! Mail: agora com 1GB de espaço grátis. Abra sua conta!




--
Use o melhor sistema de busca da Internet
Radar UOL - http://www.radaruol.com.br




=
Instruções para entrar na lista, sair da lista e usar a lista em
http://www.mat.puc-rio.br/~nicolau/olimp/obm-l.html
=


Re: [obm-l] Denovo o problema da Elipse...

2005-02-28 Por tôpico Rogerio Ponce
Ola Bruno,
 
considere que a elipse em questao tenha o semi-eixo menor igual a 1 (s.p.d.g.), e semi-eixo maior igual a "a" , e que seu centro "O" esteja na origem de um sistema cartesiano de eixos x e y , de forma que sua equacao e' x^2/a^2 + y^2 = 1 .  
Trace uma vertical pelo ponto P, e seja Q sua intersecao com o eixo x.Trace horizontais pelos pontos P e T' , e sejam R e S suas respectivas intersecoes com o eixo y.Seja U a intersecao da reta TT' com o eixo y.
Considere o plano que se obtem ao girarmos o plano xy em torno do eixo y, do angulo arccos(1/a) . Repare que a projecao da elipse sobre este plano e' uma circunferencia de raio 1. Paralelamente, obtemos triangulos semelhantes pela projecao dos triangulos URP, UST' e OPQ sobre este mesmo plano.Assim, podemos escrever as 2 relacoes[ (xQ - xO)/a ] / [ yP - yQ ]   =  [ yU - yS ] / [ (xT' - xS)/a ]
[ (xQ - xO)/a ] / [ yP - yQ ]   =  [ yU - yR ] / [ (xP - xR)/a ]
ou seja, [ xP / a ]  / yP = [ yU - yT' ] / 1
[ xP / a ]  / yP = [ yU - yP ] / [ xP / a ]
Mas, como P pertence 'a elipse,  xP ^2 / a^2 + yP ^2 = 1Dessa forma, da 2a. equacao obtemos yU = 1/yP
 
Como o foco F esta' em x = -sqrt( a^2 - 1 ) , o comprimento do segmento UF vale sqrt ( a^2 - 1 + yU^2 )
 
Entretanto o comprimento de UT'  vale
 sqrt( xT'^2 +  (yU-yT')^2 )  = sqrt( a^2 + 1/yP^2 - 1 ) = sqrt( a^2 + yU^2 - 1)
 
Portanto, como UF e UT' tem o mesmo comprimento, o ponto F pertence 'a circunferencia com diametro TT' .  Logo o angulo TFT'  e' reto.
 
[]'s
Rogerio Ponce.
 
 
 
 
 
Bruno Bonagura <[EMAIL PROTECTED]> wrote:
Estou mandando novamente um problema que mandei para a lista há um tempoatrás. Imagino que os senhores tiveram dificuldade em acessar a imagem poiso servidor do uol não permite acesso direto a arquivos de imagem.Está aqui o link do enunciado.http://cienciasexatas.sites.uol.com.br/elipse.htmGostaria de uma demonstração com uso de geometria plana. Através deanalítica eu ja consegui a prova mas gostaria muito de ter uma demonstraçãoatravés de conceitos da geometria euclidiana.Agradeço respostas!Bruno Bonagurahttp://cienciasexatas.blog.uol.com.br__Converse com seus amigos em tempo real com o Yahoo! Messenger http://br.download.yahoo.com/messenger/ 

[obm-l] Saida de emergencia

2005-02-24 Por tôpico Rogerio Ponce


Uma equipe da Boeing deseja construir uma rampa (um escorregador) para a saida de emergencia de um aviao com as seguintes caracteristicas:
- a saida de emergencia esta' a 10m de altura em relacao ao chao.
- a rampa termina a 10m de distancia da projecao vertical da porta do aviao
- a aceleracao da gravidade e'  10m/s^2 , e a rampa nao oferece atrito
 
Sabendo-se que a descida pelo escorregador deve ser feita no menor tempo possivel, e que o passageiro nao da' nenhum impulso na saida (isto e', a velocidade inicial e' zero) , qual a funcao que descreve o perfil da rampa?
 
Abracos,
Rogerio Ponce
 
		Yahoo! Acesso Grátis - Internet rápida e grátis. Instale o discador do Yahoo! agora.

Re: [obm-l] Dúvida do livro da SBM

2005-02-24 Por tôpico Rogerio Ponce
Ola' Andre' ,
a equacao do 2o. grau em (x^1/2)
(x^1/2) ^ 2   - (x^1/2)  - m = 0
tem sempre uma raiz positiva e outra negativa, se m>0 .
Considerando que x seja real, somente a raiz positiva servira', isto e' :
x^1/2 =  (1 + sqrt(1 + 4m)) / 2
 
Abraços,
Rogerio Ponce.
 
André Barreto <[EMAIL PROTECTED]> wrote:

Estou com algumas dúvidas em umas questões do livro A Matemática do Ensino Médio da SBM. Vou mandar a que tentei mais e vou guardar algumas para caso eu não consiga mesmo.
 
7. Mostre que, para todo m > 0, a equação x^1/2 + m = x tem exatamente uma raiz.
Essa eu pensei na representação destas no plano cartesiano, y = x^1/2 + m e y = x, desenhei a primeira bissetriz no plano cartesiano e se y = x^1/2 haveriam duas raizes, mas como soma com um m > 0 só há uma raiz.
Usando o formato geometrico de y = x^1/2, bláblá. Será isso plausível para mostrar? Eu posso utilizar um conhecimento previo do comportamento y = x^1/2 + m dessa expressão para mostrar??, 
 
me ajudem nessa.
 
Obrigado
 
Atenciosamente
 
André Sento Sé Barreto


Yahoo! Acesso Grátis - Internet rápida e grátis. Instale o discador do Yahoo! agora.
		Yahoo! Acesso Grátis - Internet rápida e grátis. Instale o discador do Yahoo! agora.

[obm-l] triangulo equilatero

2005-02-18 Por tôpico Rogerio Ponce
Ola' pessoal,
Seja um triangulo ABC. Marque os pontos D,E e F sobre os lados AB, BC e CA  
tal que AD=BE=CF.

Prove que se o triangulo DEF for equilatero, entao ABC e' equilatero.
[]'s
Rogerio Ponce
_
MSN Messenger: converse online com seus amigos .  
http://messenger.msn.com.br

=
Instruções para entrar na lista, sair da lista e usar a lista em
http://www.mat.puc-rio.br/~nicolau/olimp/obm-l.html
=


[obm-l] Re: [obm-l] LIMITAÇÕES TECNOLÓGICAS!

2005-02-18 Por tôpico Rogerio Ponce
Oi Bernardo, a calculadora era cientifica, se lembra?
pi * e  =   e ^ ( 1 + ln( asin(1) + asin(1))  )
[]'s
Rogerio Ponce
From: Bernardo Freitas Paulo da Costa
Bom, se você quiser multiplicar "pi" por "e" eu não vejo como fazer
isso só com o botão de adição...
--
Bernardo Freitas Paulo da Costa
On Thu, 17 Feb 2005 16:59:38 -0500, Qwert Smith <[EMAIL PROTECTED]> 
wrote:
> Depende da calculadora que vc tem...
> vc sempre pode gastar seu dedo no botao da adicao
>
> se vc tem uma calculador cientifica vc pode fazer assim:
>
> A*B = 10^[(logA)+(logB)]
>
> >From: Chicao Valadares <[EMAIL PROTECTED]>
> >
> >[snip]
> >
> >A propósito. como multiplicar dois números numa
> >calculadora cujas teclas de
> >produto e divisão estejam danificadas?
> >
> >  Abraços!
> >
>
>
_
Chegou o que faltava: MSN Acesso Grátis. Instale Já! 
http://www.msn.com.br/discador

=
Instruções para entrar na lista, sair da lista e usar a lista em
http://www.mat.puc-rio.br/~nicolau/olimp/obm-l.html
=


Re: [obm-l] Moedas em sacos

2005-02-16 Por tôpico Rogerio Ponce
Agora sim, e' 7491 mesmo !
Valeu Fabio, Fernando, Claudio, Bruno e Qwert !
[]'s
Rogerio Ponce.
From: Fábio Dias Moreira
Fábio Dias Moreira escreveu:
Rogerio Ponce escreveu:
Ola' Qwert, Bruno, Claudio e colegas da lista,
o fato e' que N pode ser ainda maior que 927...
[...]

Considere todos os ternos (p, q, r) de inteiros com |p|, |q|, |r| <=
10 e tais que mdc(p, q, r) = n (estou definindo mcd(x, 0) = |x|).
Seja S o conjunto desses ternos. Eu afirmo que é possível fazer o
pedido com N = #S.
[...]
Logo
N = #S_1 = 9261 - 1331 - 343 - 125 - 27 + 27 + 27 + 1 = 7490.
Isso também prova que, se todas as pesagens forem balanceadas, essa
*é* a cota superior, logo basta provar que pesagens não-balanceadas
não permitem ir além de um limite inferior a 7490.
[...]
Desculpem -- eu quero dizer 7491. O terno (0, 0, 0) pode ser adicionado a S 
sem risco de ser confundido com algum dos outros ternos.

[]s,
--
Fábio Dias Moreira
_
MSN Messenger: converse online com seus amigos .  
http://messenger.msn.com.br

=
Instruções para entrar na lista, sair da lista e usar a lista em
http://www.mat.puc-rio.br/~nicolau/olimp/obm-l.html
=


Re: [obm-l] Moedas em sacos

2005-02-15 Por tôpico Rogerio Ponce
Ola' Qwert, Bruno, Claudio e colegas da lista,
o fato e' que N pode ser ainda maior que 927...
[]'s
Rogerio.

From: "Qwert Smith"
Ok N=927 and counting...
_
Chegou o que faltava: MSN Acesso Grátis. Instale Já! 
http://www.msn.com.br/discador

=
Instruções para entrar na lista, sair da lista e usar a lista em
http://www.mat.puc-rio.br/~nicolau/olimp/obm-l.html
=


Re: [obm-l] Moedas em sacos

2005-02-15 Por tôpico Rogerio Ponce
Caro Claudio,
como sempre a sua engenhosidade é bem vinda.
Mas N pode ser ainda maior...
Grande abraço,
Rogério.
From: "claudio.buffara" > Ola' pessoal,
>
> Existem N sacos abertos com 10 moedas cada um.
> Um deles, defeituoso, tem 10 moedas iguais entre si, porem mais pesadas 
que
> o padrao. Os outros sacos tem as 10 moedas com o peso padrao (a 
principio
> desconhecido).
>
> Voce dispoe de uma balanca de 2 pratos, que fornece a diferenca de peso
> entre os pratos (prato da esquerda menos prato da direita).
>
> Qual o maior N que ainda permite a determinacao do saco defeituoso com
> apenas 3 leituras ?
>
> []'s
> Rogerio Ponce
>

Eu achei N = 242 mas não sei provar que este é o maior N possível.
Suponhamos que uma moeda normal pese P e uma moeda mais pesada pese P+Q.
PRIMEIRA PESAGEM:
Colocamos 121 sacos num prato e 121 no outro.
A balança indicará em que prato está o saco mais pesado e o também o valor 
de Q, igual a 1/10 da leitura da balança.

SEGUNDA PESAGEM:
Numeramos os 121 sacos que incluem o mais pesado de 0 a 120 e fazemos o 
seguinte:
Sacos 0 a 10: 0 moedas no prato da E e 10 moedas no prato da D;
Sacos 11 a 21: 1 moeda no prato da E e 9 moedas no prato da D;
...
Sacos 11k a 11k+10: k moedas no prato da E e 10-k moedas no prato da D;
...
Sacos 110 a 120: 10 moedas no prato da E e 0 moedas no prato da D.

(obs: estou supondo que mesmo após colocar as moedas nos pratos da balança, 
continuamos a saber de que saco elas vieram. Por exemplo, podemos empilhar 
as moedas de um mesmo saco e operar a balança com cuidado de forma que as 
pilhas não desabem)

Suponhamos que o número do saco mais pesado seja 11k + r (0 <= r <= 10).
Nesse caso, os pesos em cada prato serão:
E = 605P + kQ
e
D = 605P + (10-k)Q
Logo, leitura da balança = E - D = (2k-10)Q.
Como já sabemos o valor de Q, ficaremos sabendo o valor de k.
Ou seja, após esta segunda pesagem, ficaremos sabendo que o saco mais 
pesado é um dos 11 seguintes: 11k, 11k+1, ..., 11k+10.

TERCEIRA PESAGEM:
Re-numeramos os 11 sacos que incluem o mais pesado de 0 a 10 e fazemos o 
seguinte:
Saco 0: 0 moedas no prato da E e 10 moedas no prato da D;
Saco 1: 1 moeda no prato da E e 9 moedas no prato da D;
...
Saco m: m moedas no prato da E e 10-m moedas no prato da D;
...
Saco 10: 10 moedas no prato da E e 0 moedas no prato da D.

Suponhamos que o saco mais pesado seja o m-ésimo.
Os pesos em cada prato serão:
E = 55P + mQ
e
D = 55P + (10-m)Q.
Leitura da balança = E - D = (2m-10)Q.
Como conhecemos Q, podemos determinar m e acabou.
[]s,
Claudio.
_
Chegou o que faltava: MSN Acesso Grátis. Instale Já! 
http://www.msn.com.br/discador

=
Instruções para entrar na lista, sair da lista e usar a lista em
http://www.mat.puc-rio.br/~nicolau/olimp/obm-l.html
=


Re: [obm-l] Moedas em sacos

2005-02-15 Por tôpico Rogerio Ponce
Ola' Fernando,
N=27 ainda e' pouco.
Repare que vc esta' apenas usando a informacao de um dos pratos pesar mais 
que o outro, sem considerar o valor dessa diferenca, fornecido pela balanca.
O fato e' que N pode ser mais alto que 27.

[]'s
Rogerio Ponce

From: Fernando Aires
Olá,
   Não sei se meu raciocínio está correto, mas eu pensei em resolver o
problema da seguinte forma:
   Como sabemos que o saco é mais pesado, para a última medição
(terceira), no pior caso, devemos ter 3 sacos. Mediríamos dois deles
na balança, e se um for mais pesado, é este; se ambos forem iguais, o
terceiro é o saco mais pesado.
   Dito isso, na segunda (penúltima) medição, devemos medir grupos de
3 sacos. Podemos medir 3 grupos, usando a mesma lógica da última
medição. Portanto, deve chegar 9 sacos na segunda medição.
   Assim, na primeira medição, pelo mesmo raciocínio, teremos 3 grupos
de 9 sacos. Portanto, o N máximo é 27.
   Espero que esteja certo...
On Sat, 12 Feb 2005 10:57:42 -0200, Rogerio Ponce wrote:
> Ola' pessoal,
>
> Existem N sacos abertos com 10 moedas cada um.
> Um deles, defeituoso, tem 10 moedas iguais entre si, porem mais pesadas 
que
> o padrao. Os outros sacos tem as 10 moedas com o peso padrao (a 
principio
> desconhecido).
>
> Voce dispoe de uma balanca de 2 pratos, que fornece a diferenca de peso
> entre os pratos (prato da esquerda menos prato da direita).
>
> Qual o maior N que ainda permite a determinacao do saco defeituoso com
> apenas 3 leituras ?
>
> []'s
> Rogerio Ponce
_
MSN Messenger: converse online com seus amigos .  
http://messenger.msn.com.br

=
Instruções para entrar na lista, sair da lista e usar a lista em
http://www.mat.puc-rio.br/~nicolau/olimp/obm-l.html
=


[obm-l] Moedas em sacos

2005-02-12 Por tôpico Rogerio Ponce
Ola' pessoal,
Existem N sacos abertos com 10 moedas cada um.
Um deles, defeituoso, tem 10 moedas iguais entre si, porem mais pesadas que 
o padrao. Os outros sacos tem as 10 moedas com o peso padrao (a principio 
desconhecido).

Voce dispoe de uma balanca de 2 pratos, que fornece a diferenca de peso 
entre os pratos (prato da esquerda menos prato da direita).

Qual o maior N que ainda permite a determinacao do saco defeituoso com 
apenas 3 leituras ?

[]'s
Rogerio Ponce
_
MSN Messenger: converse online com seus amigos .  
http://messenger.msn.com.br

=
Instruções para entrar na lista, sair da lista e usar a lista em
http://www.mat.puc-rio.br/~nicolau/olimp/obm-l.html
=


RE: [obm-l] area

2005-01-29 Por tôpico Rogerio Ponce
Olá Carlos,
Sejam G o baricentro de ABC, e M o ponto médio de BC.
Prolongue a mediana AM , para além de BC, marcando o ponto K de tal forma 
que GM = MK.
Repare que triângulo BGK é formado por 2/3 das medianas de ABC.

Daqui pra frente você completa...
Abraços,
Rogério.

From: "carlos gomes"
Como sair dessa?
Qual a razão entra as medidas das áreas de um triângulo ABC e do triângulo 
DEF cujos lados têm como medidas as medidas das medianas do triângulo ABC?. 
Como saber se as medidas das referidas medianas, de fato formam um 
triângulo?

CGomes
_
MSN Messenger: converse online com seus amigos .  
http://messenger.msn.com.br

=
Instruções para entrar na lista, sair da lista e usar a lista em
http://www.mat.puc-rio.br/~nicolau/olimp/obm-l.html
=


Re: [obm-l] 6 casais

2005-01-26 Por tôpico Rogerio Ponce
Olá Carlos,
nem todos da lista têm o livro.
[]'s
Rogério.
Carlos Victor <[EMAIL PROTECTED]> wrote:
Olá ,Esta questão está resolvida na página 98 do livro de J.Plínio O. Santos : " Introdução `a análise Combinatória ".[]´s Carlos VictorAt 18:03 20/1/2005, Rogerio Ponce wrote:>Olá pessoal, esse aqui lembra outro bem fácil:>>De quantas formas podemos acomodar 6 casais ao redor de uma mesa redonda, >de forma que nenhum marido fique ao lado de sua esposa?>>Atenção distraídos : são DOZE pessoas.>>Abraços a todos,>Rogério.>>_>MSN Messenger: converse online com seus amigos .>http://messenger.msn.com.br>>=>Instruções para entrar na lista, sair da lista e usar a lista
 em>http://www.mat.puc-rio.br/~nicolau/olimp/obm-l.html>==Instruções para entrar na lista, sair da lista e usar a lista emhttp://www.mat.puc-rio.br/~nicolau/olimp/obm-l.html=
		Yahoo! Acesso Grátis - Internet rápida e grátis. Instale o discador do Yahoo! agora.

[obm-l] 6 casais

2005-01-20 Por tôpico Rogerio Ponce
Olá pessoal, esse aqui  lembra outro bem fácil:
De quantas formas podemos acomodar 6 casais ao redor de uma mesa redonda, de 
forma que nenhum marido fique ao lado de sua esposa?

Atenção distraídos : são DOZE pessoas.
Abraços a todos,
Rogério.
_
MSN Messenger: converse online com seus amigos .  
http://messenger.msn.com.br

=
Instruções para entrar na lista, sair da lista e usar a lista em
http://www.mat.puc-rio.br/~nicolau/olimp/obm-l.html
=


RE: [obm-l] ENUNCIADO ORIGINAL!

2005-01-20 Por tôpico Rogerio Ponce
Olá João,
você não leu a frase inteira.
Eu respondi "dois pares IGUAIS e um diferente" .
Portanto não existe o "C" da sua configuração.
Ou temos "AA, AA e BB", com todos os pares casados,
ou então temos "AB, AA e BA", com apenas um par casado.
Deixo para você a demonstração de que esta é a única resposta correta.
Dica: repare que a pergunta usa claramente um "ou" exclusivo, eliminando 
portanto a sua resposta.

Abraços,
Rogério.
From: João Gilberto Ponciano Pereira
Hum... eu diria que é apenas um par, dois sapatos.
3 pares podem fazer a configuração AB, BC, CA.
-Original Message--
From: Rogerio Ponce
Olá Jorge,
de estalo eu diria que são 3 pares: 2 iguais e 1 diferente.
Seis sapatos, portanto.
Abraços,
Rogério.
>From: jorgeluis
>Date: Wed, 19 Jan 2005 21:25:00 -0300
>
>Num hotel, os sapatos a serem engraxados são colocados do lado de fora da
>porta.
>A camareira que realiza este trabalho tem que responder à pergunta sobre
>quantos
>sapatos há a limpar. Então diz: "Se reunirmos os sapatos em pares, 
combinam

>ou
>um só par ou todos os pares". Quantos são os sapatos?
>
_
MSN Hotmail, o maior webmail do Brasil.  http://www.hotmail.com
=
Instruções para entrar na lista, sair da lista e usar a lista em
http://www.mat.puc-rio.br/~nicolau/olimp/obm-l.html
=


RE: [obm-l] ENUNCIADO ORIGINAL!

2005-01-20 Por tôpico Rogerio Ponce
Olá Jorge,
de estalo eu diria que são 3 pares: 2 iguais e 1 diferente.
Seis sapatos, portanto.
Abraços,
Rogério.
From: jorgeluis
Date: Wed, 19 Jan 2005 21:25:00 -0300
Num hotel, os sapatos a serem engraxados são colocados do lado de fora da 
porta.
A camareira que realiza este trabalho tem que responder à pergunta sobre 
quantos
sapatos há a limpar. Então diz: "Se reunirmos os sapatos em pares, combinam 
ou
um só par ou todos os pares". Quantos são os sapatos?

_
MSN Hotmail, o maior webmail do Brasil.  http://www.hotmail.com
=
Instruções para entrar na lista, sair da lista e usar a lista em
http://www.mat.puc-rio.br/~nicolau/olimp/obm-l.html
=


Re: [obm-l] Teoria do Caos & Problema P vs. NP

2005-01-20 Por tôpico Rogerio Ponce
Olá Ivan,
acho que você tem muito o que conversar com o Fabiano Sutter, antigo 
participante aqui da lista, que já deve ter quebrado algumas chaves da RSA à 
essa altura.
Se a CIA ainda não o levou, procure-o urgentemente - vocês falam a mesma 
língua, e parece que ele está trabalhando exatamente com isso: formas gerais 
de relação detalhadamente simples, que explicam problemas que têm a ver com 
teorias (genéricas ou específicas) locais e globalizadas.
Obviamente isso se aplica diretamente à relação entre a Teoria do Caos e o 
Problema P vc. NP .
Espero ter ajudado.

Grande abraço a todos,
Rogério.

From: "Ivan Miranda" Subject: [obm-l] Teoria do Caos & Problema P vs. NP
Date: Tue, Jan 18, 2005 09:41:45AM -0300
Pessoal,
Desculpem-me por não falar as minhas intenções no último problema enviado,
mas esqueci. Só queria saber, supondo que aquele número pudesse existir (me
refiro ao "0,000...01 é diferente de 0?"), e realmente havia infinitos 
zeros
"antes" do 1. Espero não ter causado problemas.
Mas peço que respondam a essa minha pergunta com qualquer tipo de resposta
(detalhada ou simples), porém que diga algo sobre e os englobe de uma forma
geral de relação: isso do problema pode ser explicado com aquilo, o 
problema
não pode ser resolvido sem a teoria porque..., a teoria tem tudo a ver com 
o
problema, porque..., entre outros (esses exemplos são só para ilustrar e
podem estar errados).

Qual a relação da Teoria do Caos com o Problema P vs. NP?
Desculpem qualquer coisa e obrigado pela atenção,
Ivan Miranda.
_
MSN Messenger: converse online com seus amigos .  
http://messenger.msn.com.br

=
Instruções para entrar na lista, sair da lista e usar a lista em
http://www.mat.puc-rio.br/~nicolau/olimp/obm-l.html
=


[obm-l] [obm-l] Probabilidade em amigo oculto - SOLUÇÃO

2005-01-08 Por tôpico Rogerio Ponce
Olá pessoal,
qual a probabilidade P(N) de ocorrer um sorteio válido numa reunião de N 
"amigos ocultos" ?
(sorteio válido é aquele em que ninguém sorteia a si mesmo).

-
Primeiramente, em um sorteio qualquer, existem sub-grupos do tipo "A sorteia 
B, que sorteia C, que sorteia...que sorteia A" , formando um 'loop'. 
Chamemos de 'cadeia' essa sequência de pessoas.

Então, seja V(n) o número de sorteios válidos com 'n' pessoas.
Quando acrescentamos a enésima-primeira pessoa a um grupo com 'n' pessoas, 
um sorteio válido qualquer corresponderá às seguintes situações:

a) essa pessoa forma uma cadeia com mais de 2 elementos.
b) essa pessoa forma uma cadeia com apenas 2 elementos (ela e uma 2a. pessoa 
fazem uma troca mútua de presentes).

No caso 'a', podemos considerar que essa pessoa é "inserida" em alguma das 
cadeias que haveria num sorteio válido com apenas 'n' pessoas.
No caso 'b' , cada sorteio pode ser obtido a partir da escolha do 2o. 
elemento, e então formando-se todos os sorteios válidos possíveis com (n-1) 
elementos.

Dessa forma, o número de sorteios válidos do tipo 'a' vale 'n*V(n)' .
Repare que essa nova pessoa pode ser inserida logo após uma pessoa qualquer 
dentre as 'n' existentes.

E o número de sorteios válidos do tipo'b' vale 'n*V(n-1)' .
Repare que essa nova pessoa pode fazer par com qualquer uma dentre as 'n' 
existentes, enquanto as outras (n-1) se organizam como um sorteio válido de 
(n-1) elementos.

Assim, V(n+1) = n*V(n) + n*V(n-1)
Fazendo V(n) = n! * W(n) , obtemos a equação de diferenças, linear e 
homogêna, do 1o grau:
[W(n+1) - W(n)]  +  1/(n+1) * [W(n) - W(n-1)]  = 0

Portanto, a solução geral é
W(n+1) - W(n) = C * (-1)^(n+1)/(n+1)!
Como V(1)=0 e V(2)=1 , então C=1 , pois W(1)=0 e W(2)=1/2 , que nos leva a
W(n+1) = W(n) + (-1)^(n+1)/(n+1)!
Como o número de sorteios possíveis é n! , a probabilidade de sorteios 
válidos com 'n' pessoas é P(n)= V(n)/n! .
Logo, P(n) = W(n) , ou seja,

P(n) = P(n-1) + (-1)^n/n! , onde P(1)=0
Além disso, é fácil verificar que quando 'n' cresce, P(n) converge para
P = 0 +1/2! - 1/3! + 1/4! ... = 1/e
[]'s,
Rogério.
_
MSN Hotmail, o maior webmail do Brasil.  http://www.hotmail.com
=
Instruções para entrar na lista, sair da lista e usar a lista em
http://www.mat.puc-rio.br/~nicolau/olimp/obm-l.html
=


RE: [obm-l] Problemas em aberto

2005-01-08 Por tôpico Rogerio Ponce
1) Construir uma estrutura rígida usando apenas três varetas rígidas de
mesmo comprimento e barbante, de modo que duas varetas quaisquer não se
toquem.
3) Decomponha o numero real positivo A numa soma de parcelas positivas:
x_1 + x_2 + ... + x_r = A
de forma que o produto x_1*x_2*...*x_r seja o maior possivel.
(ninguem falou que as parcelas precisam ser inteiras)
Caro Claudio,
só você para me fazer cortar fios de cobre e pedaços de barbante num dia 
como esse...
Mas é possível!
Não ficou muito bonito (a obra está claramente mais ligada a OBM que ao 
MAM), mas dá pra se entender... o resultado é um octaedro não-regular, em 
que as varetas são as diagonais internas, e as arestas são de barbante.

Já o (3) faz parte da solução do ¨água e cachaça¨, que propus há muito. Era 
essa a idéia?

Em relação ao (6): ainda hoje escrevo a solução combinatória do ¨amigo 
oculto¨, ok?

Grande abraço e ótimo 2005!
Rogério.
PS: e o chopp com o Prof. Morgado?
_
MSN Messenger: converse online com seus amigos .  
http://messenger.msn.com.br

=
Instruções para entrar na lista, sair da lista e usar a lista em
http://www.mat.puc-rio.br/~nicolau/olimp/obm-l.html
=


RE: [obm-l] Problema de natural

2005-01-08 Por tôpico Rogerio Ponce
Olá André,
o mínimo é 7, que vc também poderia obter fazendo y=200, z=9 e x=5 .
E além dessas, não existem outras combinações que levem ao mínimo de 7.
(isso fica pra você mesmo provar. Dica: 7<= Log(y) + x*Log(z) < 8 )
[]'s
Rogério.
From: André Barreto
Como eu posso provar que esse é o resultado correto? Ou pelo menos que não 
existem outros?

Rogerio Ponce wrote:
Olá André,
se entendi o que você pediu, o resultado de "y * z^x" deve ter 8 
algarismos,
e a soma dos algarismos de y e x deve ser mínima.
Então
1) faça z o maior possível: z=9
2) faça y o menor possível, já que o "investimento em x rende mais" (x é
expoente) : y=100
3) assim, assim x=6 (levando ao total de 53144100)

Resultado: a soma mínima é 7.
_
MSN Messenger: converse online com seus amigos .  
http://messenger.msn.com.br

=
Instruções para entrar na lista, sair da lista e usar a lista em
http://www.mat.puc-rio.br/~nicolau/olimp/obm-l.html
=


RE: [obm-l] Problema de natural

2005-01-07 Por tôpico Rogerio Ponce
Olá André,
se entendi o que você pediu, o resultado de "y * z^x" deve ter 8 algarismos, 
e a soma dos algarismos de y e x deve ser mínima.
Então
1) faça z o maior possível: z=9
2) faça y o menor possível, já que o "investimento em x rende mais" (x é 
expoente) : y=100
3) assim, assim x=6 (levando ao total de 53144100)

Resultado: a soma mínima é 7.

From: André Barreto
Do nada elaborei um problema futucando na minha calculadora. Tentei 
resolver, mas não deu... não sei nem se tem como, mas todo caso lá vai ele.


Vc possui uma calculadora que comporta números de até 8 algarismos.
Qual o número natural (y) de 3 algarismos, que multiplicado por um natural 
de 1 algarismo, x vezes alcança 8 algarismos exatamente. Sendo a soma dos 
algarismos de (y) + x o menor número possível para que isso aconteça?


Será possível resolver isso? Ou pelo menos chegar a algumas conclusões 
interessantes?

_
MSN Messenger: converse online com seus amigos .  
http://messenger.msn.com.br

=
Instruções para entrar na lista, sair da lista e usar a lista em
http://www.mat.puc-rio.br/~nicolau/olimp/obm-l.html
=


Re: [obm-l] probleminha..

2004-12-29 Por tôpico Rogerio Ponce
Olá Daniel,
é comum que se saiba que "A interpretação faz parte da questão".
Pois saiba também que excesso de imaginação vale ZERO na maioria das provas...:-)
 
Abraços,
Rogério.
 
 
"Daniel S. Braz" <[EMAIL PROTECTED]> wrote:
se as cidades estiverem todas sobre uma reta e se a estrada 11-1 forem diagonal eh possivel...algo mais ou menos assim...usem aimaginacao..hehe| r|---| 01 |---|---| 02 |---|---| 03 |---|...|---| 11 |---||As estradas 1-2; 2-3; ... ; 10-11 estao sobre a reta r e a estrada11-1 (nao desenhada) esta na diagonal..[]sdaniel
		Yahoo! Acesso Grátis - Internet rápida e grátis. Instale o discador do Yahoo! agora.

Re: [obm-l] Quadrilatero Circunscritivel

2004-12-14 Por tôpico Rogerio Ponce
Olá Luís,
É o mesmo problema, porém com o enunciado correto.
A diferença é que agora, o ponto(F) por onde passa a perpendicular está 
definido.
Conforme mostrei anteriormente, sem a amarração deste ponto o problema seria 
impossível de ser demonstrado, a não ser no caso particular em que o centro 
do círculo estivesse sobre a outra diagonal do quadrilátero.

Abraços,
Rogério.

De: Luís Lopes
Sauda,c~oes,
O 1o. enunciado tirei de uma lista. Será o mesmo
problema que o Claudio propôs (2o. enunciado) ?
Uma solução segue mais abaixo.
[]'s
Luis
Circle with center in point H is inscribed into convex quadrilateral
ABCD, point H doesn't lie on line AC. Diagonals AC and BD intersect
at point F. Line passing through point F and perpendicular to line
BD, cuts lines AH and CH in points R and S respectively. Prove that
RF=FS.
Michel Swift

Let ABCD be a quadrilateral with an inscribed circle with center O (all
sides of the quadrilateral are tangent to the circle). Assume that O does
not lie on AC. Construct a line through E on BD perpendicular to BD.
Then this line meets OA at R and OC at S. Show that |ER|=|ES|.
  One can view vertices of ABCD quadrilateral as foci and points of
  a rectangular hyperbola  ( B,D  - foci,   A,C - points on
  one branch,   because  AB+CD = BC+AD  or AB-AD = BC-CD = 2a )
  Then lines AH, CH are tangents to hyperbola at A and C.
  A line perpendicular to BD (main axis) at point F cuts tangents at
  symmetric points wrt BD.
  Cheers,
  K. Jeff.


From: Claudio Buffara <[EMAIL PROTECTED]>
Reply-To: [EMAIL PROTECTED]
To: <[EMAIL PROTECTED]>
Subject: Re: [obm-l] Quadrilatero Circunscritivel
Date: Sun, 12 Dec 2004 00:25:25 -0200
on 11.12.04 22:07, Rogerio Ponce at [EMAIL PROTECTED] wrote:
Só pra tirar as dúvidas:
Suponha que |ER| = |ES| .
Agora trace uma paralela a RS original, determinando novos pontos E,R e S.
Como o ponto O não pertence a BD, necessariamente os novos |ER| e |ES| 
serão
diferentes entre si.

Nao necessariamente. E se BD for a mediatriz de RS? Alias, isso eh
justamente o que o problema pede que se prove.

Portanto, não basta que o ponto E pertença a BD.
[]'s
Rogério.

--

Eu acho que é para qualquer E pertencente a BD... pelo menos no
desenho que eu fiz ficou assim. Mas ainda não demonstrei.

Essa tambem foi a minha interpretacao.
[]s,
Claudio.
Abraços
--
Bernardo Freitas Paulo da Costa
------
From: "Rogerio Ponce"
Olá Claudio,
onde fica o ponto E?
Com certeza, não basta ele pertencer a BD.
[]'s,
Rogério.
-
Oi, pessoal:
Um amigo me mandou este aqui, o qual nao me parece muito trivial...
Let ABCD be a quadrilateral with an inscribed circle with center O (all
sides of the quadrilateral are tangent to the circle). Assume that O does
not lie on AC. Construct a line through E on BD perpendicular to BD.
Then this line meets OA at R and OC at S. Show that |ER|=|ES|.
[]s,
Claudio.
_
Natal no MSN Shopping: COMPROU, GANHOU $$! Veja Como! 
http://shopping.msn.com.br/MSNSHopping/GuiaEspeciais/Natal/conteudo.aspx?cd_guia=20&cd_funcao=238

=
Instruções para entrar na lista, sair da lista e usar a lista em
http://www.mat.puc-rio.br/~nicolau/olimp/obm-l.html
=


Re: [obm-l] Quadrilatero Circunscritivel

2004-12-11 Por tôpico Rogerio Ponce
Só pra tirar as dúvidas:
Suponha que |ER| = |ES| .
Agora trace uma paralela a RS original, determinando novos pontos E,R e S.
Como o ponto O não pertence a BD, necessariamente os novos |ER| e |ES| serão 
diferentes entre si.

Portanto, não basta que o ponto E pertença a BD.
[]'s
Rogério.

--

Eu acho que é para qualquer E pertencente a BD... pelo menos no
desenho que eu fiz ficou assim. Mas ainda não demonstrei.

Abraços
--
Bernardo Freitas Paulo da Costa
--
From: "Rogerio Ponce"
Olá Claudio,
onde fica o ponto E?
Com certeza, não basta ele pertencer a BD.
[]'s,
Rogério.
-
Oi, pessoal:
Um amigo me mandou este aqui, o qual nao me parece muito trivial...
Let ABCD be a quadrilateral with an inscribed circle with center O (all
sides of the quadrilateral are tangent to the circle).  Assume that O does
not lie on AC.  Construct a line through E on BD perpendicular to BD.
Then this line meets OA at R and OC at S.  Show that |ER|=|ES|.
[]s,
Claudio.
_
MSN Messenger: converse com os seus amigos online.  
http://messenger.msn.com.br

=
Instruções para entrar na lista, sair da lista e usar a lista em
http://www.mat.puc-rio.br/~nicolau/olimp/obm-l.html
=


Re: [obm-l] x^2=2^x - Como Resolver?

2004-12-11 Por tôpico Rogerio Ponce
Olá Rafael,
infelizmente está errado.
Observe que derivar a equação significa igualar a derivada das duas funções, 
o que não faz o menor sentido, pois vc está procurando a igualdade de 
valores, e não a mesma inclinação de tangentes.

[]'s
Rogério.

De: Faelccmm
Assunto: Re: [obm-l] x^2=2^x - Como Resolver?
Data: Fri, 10 Dec 2004 18:50:54 -0800

Olá !
2^x = x^2
Derivando os dois membros:
d(2^x)/dx = d(x^2)x
x*(2^(x-1)) = 2*(x^(2-1))
x*(2^(x-1)) = 2*x
Eliminando x:
(2^(x-1)) = 2
x - 1 = 1
x = 2
Está certo isso ?
[]s,
Rafael
_
MSN Messenger: converse com os seus amigos online.  
http://messenger.msn.com.br

=
Instruções para entrar na lista, sair da lista e usar a lista em
http://www.mat.puc-rio.br/~nicolau/olimp/obm-l.html
=


Re: [obm-l] Quadrilatero Circunscritivel

2004-12-11 Por tôpico Rogerio Ponce
Olá Claudio,
onde fica o ponto E?
Com certeza, não basta ele pertencer a BD.
[]'s,
Rogério.
-
Oi, pessoal:
Um amigo me mandou este aqui, o qual nao me parece muito trivial...
Let ABCD be a quadrilateral with an inscribed circle with center O (all
sides of the quadrilateral are tangent to the circle).  Assume that O does
not lie on AC.  Construct a line through E on BD perpendicular to BD.
Then this line meets OA at R and OC at S.  Show that |ER|=|ES|.
[]s,
Claudio.
_
Natal no MSN Shopping: COMPROU, GANHOU $$! Veja Como! 
http://shopping.msn.com.br/MSNSHopping/GuiaEspeciais/Natal/conteudo.aspx?cd_guia=20&cd_funcao=238

=
Instruções para entrar na lista, sair da lista e usar a lista em
http://www.mat.puc-rio.br/~nicolau/olimp/obm-l.html
=


Re: [obm-l] x^2=2^x - Como Resolver?

2004-12-11 Por tôpico Rogerio Ponce
Olá Rafael,
infelizmente está errado.
Observe que derivar a equação significa igualar a derivada das duas funções, 
o que não faz o menor sentido, pois vc está procurando a igualdade de 
valores, e não a mesma inclinação de tangentes.

[]'s
Rogério.

De: Faelccmm
Assunto: Re: [obm-l] x^2=2^x - Como Resolver?
Data: Fri, 10 Dec 2004 18:50:54 -0800

Olá !
2^x = x^2
Derivando os dois membros:
d(2^x)/dx = d(x^2)x
x*(2^(x-1)) = 2*(x^(2-1))
x*(2^(x-1)) = 2*x
Eliminando x:
(2^(x-1)) = 2
x - 1 = 1
x = 2
Está certo isso ?
[]s,
Rafael
_
Natal no MSN Shopping: COMPROU, GANHOU $$! Veja Como! 
http://shopping.msn.com.br/MSNSHopping/GuiaEspeciais/Natal/conteudo.aspx?cd_guia=20&cd_funcao=238

=
Instruções para entrar na lista, sair da lista e usar a lista em
http://www.mat.puc-rio.br/~nicolau/olimp/obm-l.html
=


RE: [obm-l] lancamento de dados

2004-12-02 Por tôpico Rogerio Ponce
Olá pessoal,
como falei antes, a expressão final é um montrengo: é a integral de uma 
expressão com apenas 2047 termos exponenciais.
Após breve cálculo, o resultado que se obtém é 769767316159 / 12574325400 , 
que dá aproximadamente 61.2 lançamentos.

Para quem se interessar pelo assunto, a referência é o "coupon collector 
problem", facilmente encontrável na web.

[]'s
Rogério.

From: "Rogerio Ponce"
Olá pessoal,
utilizando-se dois dados, qual o número médio de lançamentos duplos para 
obtermos todas as somas possíveis?

[]'s
Rogério.
_
Natal no MSN Shopping: COMPROU, GANHOU $$! Veja Como! 
http://shopping.msn.com.br/MSNSHopping/GuiaEspeciais/Natal/conteudo.aspx?cd_guia=20&cd_funcao=238

=
Instruções para entrar na lista, sair da lista e usar a lista em
http://www.mat.puc-rio.br/~nicolau/olimp/obm-l.html
=


[obm-l] Re:[obm-l] Eq. Logarítmica (volta)

2004-12-02 Por tôpico Rogerio Ponce
Olá Osvaldo,
o que talvez sua prima tenha querido dizer com "solução analítica" é que o 
problema admite "solução não numérica".

Abraços,
Rogério.
---
De: Osvaldo Mello Sponquiado
Assunto: [obm-l] Eq. Logarítmica (volta)
Data: Thu, 02 Dec 2004 11:03:55 -0800
Minha prima disse que EXISTE solução analítica para este problema. Ele 
intrigou muitos professores da cidade dela, até chegou a ser desafio. Peço 
a ajuda do prof. Nicolau com ele.
_
MSN Hotmail, o maior webmail do Brasil.  http://www.hotmail.com
=
Instruções para entrar na lista, sair da lista e usar a lista em
http://www.mat.puc-rio.br/~nicolau/olimp/obm-l.html
=


RE: [obm-l] lancamento de dados

2004-11-25 Por tôpico Rogerio Ponce
Desculpe Claudio,
mas acho que a solução é feia pra caramba...
Achei que seriam 18 lançamentos, que corresponde ao número médio de 
lançamentos para se obter o resultado de menor probabilidade ( 1+1 ou 6+6) .
Mas é óbvio que não posso considerar apenas o resultado para o lançamento 
menos provável.

Vou pensar mais um pouquinho...:-)
Abraços,
Rogério.

Olá Claudio,
não acho que você leve 10 minutos para pensar e resolver esse daí...
( também pensei rapidamente a respeito dele, e me pareceu bonitinho )
Abraços,
Rogério.
Olá pessoal,
utilizando-se dois dados, qual o número médio de lançamentos duplos para 
obtermos todas as somas possíveis?

[]'s
Rogério.
_
MSN Messenger: converse com os seus amigos online.  
http://messenger.msn.com.br

=
Instruções para entrar na lista, sair da lista e usar a lista em
http://www.mat.puc-rio.br/~nicolau/olimp/obm-l.html
=


RE: [obm-l] lancamento de dados

2004-11-25 Por tôpico Rogerio Ponce
Olá Claudio,
não acho que você leve 10 minutos para pensar e resolver esse daí...
( também pensei rapidamente a respeito dele, e me pareceu bonitinho )
Abraços,
Rogério.
Olá pessoal,
utilizando-se dois dados, qual o número médio de lançamentos duplos para 
obtermos todas as somas possíveis?

[]'s
Rogério.
_
MSN Hotmail, o maior webmail do Brasil.  http://www.hotmail.com
=
Instruções para entrar na lista, sair da lista e usar a lista em
http://www.mat.puc-rio.br/~nicolau/olimp/obm-l.html
=


[obm-l] lancamento de dados

2004-11-23 Por tôpico Rogerio Ponce
Olá pessoal,
utilizando-se dois dados, qual o número médio de lançamentos duplos para 
obtermos todas as somas possíveis?

[]'s
Rogério.
_
MSN Messenger: converse com os seus amigos online.  
http://messenger.msn.com.br

=
Instruções para entrar na lista, sair da lista e usar a lista em
http://www.mat.puc-rio.br/~nicolau/olimp/obm-l.html
=


RE: [obm-l] Cubo Remontado

2004-11-12 Por tôpico Rogerio Ponce
Olá Claudio,
de um total de t=n^3 cubinhos, v=8 deles pertencem aos vertices,  a=12*(n-2) 
às arestas,
e f=6*(n-2)^2 às faces. O resto, r=(n-2)^3 pertence ao 'miolo' , e não é 
visível.

A probabilidade de selecionarmos os cubinhos tipo 'vertice' corretos é :
 v! * (t-v)! / t!
A probabilidade de selecionarmos os cubinhos tipo 'aresta' corretos é :
 a! * (t-v-a)! / (t-v)!
A probabilidade de selecionarmos os cubinhos tipo 'face' corretos é :
 f! * (t-v-a-f)! / (t-v-a)!
Além disso, a probabilidade dos cubinhos tipo vértice estarem com a 
orientação correta é
(1/8) ^ v  (isto é, o único vértice "certo" do cubinho tem que estar "para 
fora" )

Também, as arestas precisam estar com a orientação correta, com a 
probabilidade de
(1/12) ^ a   (isto é, a única aresta correta para fora)

Da mesma forma, as faces precisam estar 'para fora' , com a probabilidade de
(1/6) ^ f
Multiplicando tudo, obtemos o resultado
v! * a! * f! * r!  /  [ t! * 8^v *12^a * 6^f ]
Talvez haja uma forma mais bonitinha de expressá-lo...
Grande abraço e bom fim de semana a todos!
Rogério.


From: Claudio Buffara
Um cubo, originalmente feito de madeira branca, tem suas faces pintadas de
preto. Em seguida, este cubo eh serrado de forma a produzir 27 cubinhos,
todos com o mesmo volume, os quais sao devidamente "embaralhados".
Finalmente, os 27 cubinhos sao reagrupados no escuro de forma a produzir um
novo cubo de mesmo volume que o original.
1) Qual a probabilidade deste cubo remontado ter todas as faces pretas?
(faca a hipotese obvia sobre equiprobabilidade)
2) Generalize para o caso de n^3 cubinhos.
[]s,
Claudio.
_
MSN Hotmail, o maior webmail do Brasil.  http://www.hotmail.com
=
Instruções para entrar na lista, sair da lista e usar a lista em
http://www.mat.puc-rio.br/~nicolau/olimp/obm-l.html
=


[obm-l] RE: [obm-l] DÚVIDAS!

2004-11-12 Por tôpico Rogerio Ponce
Olá Jorge!
O problema termina com a frase "não receberei a nota de 10 reais" !
Se o avô não der nota alguma, a frase seria verdadeira, causando uma 
contradição.
Se o avor der a nota de 10 reais, a frase seria falsa, causando uma 
contradição.
A única alternativa para o avô é entregar a nota de 100 reais (maximizando o 
ganho do neto!).

O trecho problemático do contra-exemplo segue abaixo:
Vovô não vai me dar a nota de cem reais. Vejam o que acontece. A frase
não pode ser falsa. Se o fosse, Pedro não poderia receber nada e a 
afirmação
passava a ser verdadeira - uma contradição. Contudo, a frase pode 
perfeitamente
ser verdadeira. Basta que o neto receba a nota de cem reais. O avô não tem 
outra
alternativa senão dar-lhe a nota de maior valor.
Está errado, pois a alternativa que torna a frase verdadeira é o avô dar a 
nota de 10 reais!

Abraços,
Rogério.

From: jorgeluis
Meus Amigos! Me ajudem a encontrar o erro neste contra-exemplo abaixo. 
Grato!

Pedro, meu neto, gosta muito de quebra-cabeças e problemas outros de 
desafios.
No dia do seu aniversário mandei chamá-lo e, para testá-lo em lógica
matemática, coloquei em cima da mesa uma nota de 10 reais e outra de 100 
reais.
"Aqui tens meu presente de aniversário. Se fizeres uma declaração 
verdadeira
dou-te uma das notas, mas se mentires não recebes nada." O pedro, após 
alguns
momentos de reflexão, concluiu que é muito fácil ganhar uma das notas. Mas 
o
legal é ter certeza de receber a nota mais valiosa. Que frase deve dizer o
Pedro para garantir que o avô lhe dará a nota de 100 reais?

Resolução: É claro que Pedro facilmente ganharia uma das notas - bastava 
dizer
uma frase verdadeira, tipo "não receberei a nota de 10 reais".

Mas ele pode
realmente, ir mais longe e obrigar o avô, mesmo contra a vontade deste, a
dar-lhe a nota de maior valor. Para isso terá de dizer uma frase
obrigatoriamente verdadeira mas que não dê alternativa ao avô. Vejamos 
algumas
hipóteses:

1) Pedro afirma, por exemplo: - Vovô vai me dar a nota de cem reais. Neste 
caso,
o avô tem duas alternativas: ou lhe diz que a frase é falsa e a prova é que 
ele
não vai receber nada; ou é de muito bom coração e considera a frase 
verdadeira,
dando-lhe os 100 reais. Pedro fica dependendo da boa vontade do avô.

2) O raciocínio é idêntico se ele afirma: - Vou receber a nota de cem, só 
lhe
podia dar 1 real.

3) Outra frase, poderia ser: - Vou ganhar as duas notas. Aqui a situação 
piora
muito. Esta frase nunca pode ser verdadeira porque o avô tinha dito que lhe
dava uma das notas e nunca duas. Neste caso Pedro nada receberia. Esgotadas 
as
frases em que Pedro diz que o avô lhe dará, é preciso analisar agora as
afirmações sobre o que o avô não lhe dará. Então facilmente se chegará à 
frase
solução: Vovô não vai me dar a nota de cem reais. Vejam o que acontece. A 
frase
não pode ser falsa. Se o fosse, Pedro não poderia receber nada e a 
afirmação
passava a ser verdadeira - uma contradição. Contudo, a frase pode 
perfeitamente
ser verdadeira. Basta que o neto receba a nota de cem reais. O avô não tem 
outra
alternativa senão dar-lhe a nota de maior valor.

OBS: Encontrada a solução, podemos ver que Pedro poderia, se quisesse 
deixar o
avô sem saber o que fazer. Bastava-lhe ter dito: - Vovô não me vai dar nota
nenhuma. Esta frase não pode ser verdadeira. Se o fosse, o neto receberia 
uma
das notas e haveria contradição. Também não pode ser falsa. Se o fosse, o 
avô
não lhe poderia dar nada e a afirmação passava a verdadeira. Nova 
contradição.

Um abraço à todos!

__
WebMail UNIFOR - http://www.unifor.br.
=
Instruções para entrar na lista, sair da lista e usar a lista em
http://www.mat.puc-rio.br/~nicolau/olimp/obm-l.html
=
_
MSN Hotmail, o maior webmail do Brasil.  http://www.hotmail.com
=
Instruções para entrar na lista, sair da lista e usar a lista em
http://www.mat.puc-rio.br/~nicolau/olimp/obm-l.html
=


[obm-l] Re: [obm-l] Combinatória

2004-11-11 Por tôpico Rogerio Ponce
Olá Maurizio,
eram 3 cores diferentes.
Dê uma olhada na mensagem do Morgado+Faelccmm (17 de setembro de 2004) em
  [EMAIL PROTECTED]/msg23407.html
Abraços,
Rogério.
-
De: Maurizio
Assunto: [obm-l] Combinatória
Bom dia
Há algum tempo lembro de ter visto na lista um problema de combinatória. To 
já faz mais de uma hora procurando aqui nos meus e-mails e não achei. Então 
vou por aqui e se alguém conseguir me enviar a solução ou resolver eu 
ficaria muito grato.

"De quantas maneiras podemos pintar as arestas de um poligono de 10 lados 
podendo usar 4 cores diferentes sem que 2 cores iguais se toquem?"

(não me lembro se o problema original era 3 ou 4 cores diferentes... creio q 
seja com 4) A resposta dava 1000 e alguma coisa...

Obrigado
Maurizio Casalaspro
_
MSN Messenger: converse com os seus amigos online.  
http://messenger.msn.com.br

=
Instruções para entrar na lista, sair da lista e usar a lista em
http://www.mat.puc-rio.br/~nicolau/olimp/obm-l.html
=


Re: [obm-l] O PROBLEMA DE JOSEFUS!

2004-11-10 Por tôpico Rogerio Ponce
Olá Felipe,
existe algum problema com sua solução.
Suponhamos seis prisioneiros, com Josefus em quarto lugar.
Para n=6 e J=4 , obtemos q=1 .
Assim, pela ordem de eliminação, sairiam os prisioneiros 1,2,3,4 e 5,
sobrando o sexto, que certamente ficaria muito agradecido a Josefus...
[]'s
Rogério.

From: Felipe Rangel
Ola Jorge e demais colegas,
Essa questao do josefus tem uma resposta muito elegante:
Josefus podera sempre se safar se ele escolher q da maneira mostrada 
abaixo:
Seja 2^x a unica potencia de 2 pertencente no intervalo n/2<=2^x<=n-1,
J>=2*(n- 2^x) implica q=J - 2*(n- 2^x) + 1,
J<2*(n- 2^x) implica q=J - 2*(n- 2^x) + 1 + n,

Eu nao vou mostrar a justificativa, so para incentivar mais pessoas a 
responder.

Sds, Felipe Rangel.
[EMAIL PROTECTED] wrote:
Ok! Pessoal! Vejam uma variante de um problema antigo em homenagem a 
Flavius
Josefus, um historiador famoso do primeiro século. Segundo a lenda, Josefus 
não
teria sobrevivido para ficar famoso se não fosse seu talento matemático. 
Durante
a guerra entre judeus e romanos, ele estava entre 11 rebeldes judeus
encurralados em uma caverna pelos romanos. Preferindo o suicídio à captura, 
os
rebeldes decidiram formar um círculo e, contando ao longo deste, matar cada
terceira pessoa restante até não sobrar ninguém. Mas Josefus, junto com um
co-conspirador não identificado, não queria saber deste pacto suicida; 
então
calculou rapidamente onde ele e seu amigo deveriam ficar neste círculo 
maligno.

Na nossa variação, começamos com n pessoas numeradas de 1 a n em um círculo 
e
eliminamos cada segunda pessoa restante até sobrar uma única pessoa. 
Suponha
que Josefus se encontra em uma determinada posição J, mas tem a chance de 
dizer
qual é o parâmetro de eliminação q tal que toda q-ésima pessoa é executada. 
Ele
sempre pode se salvar?

Vocês sabiam...que o quadrado de um número inteiro não pode terminar em 
mais de
três algarismos iguais a 4...

Abraços!
_
MSN Hotmail, o maior webmail do Brasil.  http://www.hotmail.com
=
Instruções para entrar na lista, sair da lista e usar a lista em
http://www.mat.puc-rio.br/~nicolau/olimp/obm-l.html
=


[obm-l] RE: [obm-l] A FRASE SOLUÇÃO!

2004-11-09 Por tôpico Rogerio Ponce
Olá Jorge, e colegas da lista!
Caro Jorge, acho que você se enganou no problema do Felipe. O Qwert Smith 
resolveu corretamente em 
http://www.mail-archive.com/obm-l@mat.puc-rio.br/msg25765.html

Mas vamos ao problema dos alunos:
o fator 12 não pode estar errado , pois é múltiplo de 4 e 3.
o fator 10 não pode estar errado, pois é múltiplo de 2 e 5.
11 e 13 também estão OK, pois foram eliminados pelo 10 e 12.
Também 2,3,4,5 e 6 estão OK, pois foram eliminados por 10 e 12.
Sobram 7, 8 e 9.
Se fossem 8 e 9 os fatores consecutivos, o número seria 3*4*5*7*11*13 = 
60060 > 5.
Logo, só podem ser 7 e 8 os fatores , e o número é 4*5*9*11*13 = 25740 .

Grande abraço a todos,
Rogério.



From: jorgeluis
Valeu Cláudio! Grato pela engenhosa resolução do problema do tijolo 
(CAMPEÃO!).
Quanto ao problema do Felipe, a frase solução é a seguinte: "Você não vai 
me
dar a nota de cem reais". A frase não pode ser falsa. Se o fosse, Felipe 
não
poderia receber nada e a afirmação passava a ser verdadeira - uma 
contradição.
Contudo, a frase pode perfeitamente ser verdadeira. Basta que Felipe receba 
a
nota de cem reais. Daí, eu não tenho outra alternativa senão dar-lhe a nota 
de
maior valor..

A propósito! o que Felipe poderia ter dito para me deixar sem nenhuma 
opção?

Numa classe com 12 alunos, o professor escreveu na lousa um número natural 
menor
que 50.000 e pediu que os alunos falassem alguma coisa a respeito dele. O
primeiro aluno disse que o número era múltiplo de 2, o segundo disse que o
número era múltiplo de 3 e assim sucessivamente até o último, que disse que 
o
número era múltiplo de 13. Em seguida o professor disse que, com exceção de
dois alunos consecutivos que erraram, todos os demais acertaram. Quais 
foram os
alunos que erraram? Qual foi o número que o professor escreveu?

Um abraço à todos!
_
MSN Hotmail, o maior webmail do Brasil.  http://www.hotmail.com
=
Instruções para entrar na lista, sair da lista e usar a lista em
http://www.mat.puc-rio.br/~nicolau/olimp/obm-l.html
=


Re: [obm-l] Cachorrada (probabilidade)

2004-10-22 Por tôpico Rogerio Ponce
Olá Jozias,
entre machos(M) e fêmeas(F), você poderia ter uma das seguintes combinações:
 MM
 MF
 FM
 FF
Entretanto, é dito que FF não ocorre.
Portanto a probabilidade de ambos serem machos é de 1/3.
E, ainda dentro do seu universo MM/MF/FM,
a probabilidade de se sortear um macho é 4/6, ou 2/3.
[]'s
Rogério.

De: Jozias Del Rios (ToniK)
Tenho dois cachorros. Pelo menos um deles é macho. Qual a
probabilidade dos dois serem machos? (ok...)
Se agora escolhermos um dos cachorros ao acaso, qual a
probabilidade dele ser macho?
Algum de vocês ja tentou escrever o polinomio "ba^2 + cb^2 +
ac^2" em função das funções simétricas elementares (a+b+c),
(ab+bc+ca), (abc) ?
O que eh "sagita" ??
[]'s
_
MSN Messenger: converse com os seus amigos online.  
http://messenger.msn.com.br

=
Instruções para entrar na lista, sair da lista e usar a lista em
http://www.mat.puc-rio.br/~nicolau/olimp/obm-l.html
=


RE: [obm-l] O PROBLEMA DO EXAME DE SANGUE!

2004-09-29 Por tôpico Rogerio Ponce
Olá Jorge e colegas da lista!
Essa questão de otimizar o número de exames de sangue está "pedindo" para 
ser resolvida com pesquisa binária.
O ideal é que K seja potência de 2, e que os exames sejam aplicados ao grupo 
inteiro, à metade do grupo, à quarta parte, etc..., de forma a sempre 
eliminar a metade do grupo restante.

E a probabilidade de que o teste para K pessoas seja positivo é
1 - (1-p)^K
Abraços,
Rogério.
--- from: jorgeluis -
Um grande número, N de pessoas é submetido a um exame de sangue. Este pode 
ser
efetuado de duas maneiras, (i) cada pessoa pode ser testada separadamente,
neste caso, são necessários N testes; (ii) as amostras de sangue, de K 
pessoas
podem ser misturadas e analisadas em conjunto. Se o teste é negativo, esse
único teste é suficiente para as K pessoas. Se o teste é positivo, cada uma 
das
K pessoas deve ser testada separadamente, e ao todo K + 1 testes são 
necessários
para as K pessoas. Suponha que a probabilidade p de que o teste seja 
positivo
seja a mesma para todas as pessoas e que estas sejam estocásticamente
independentes. a) Qual é a probabilidade de que o teste para uma amostra
misturada de K pessoas seja positivo? b) Qual é o valor esperado do número, 
X,
de testes necessários, sob o plano (ii)? c) Determine uma equação para o 
valor
de K que minimize o número esperado de testes sob o segundo plano. (Não 
tente
soluções numéricas) d) Mostre que esse K está próximo de 1/p^1/2 e, então, 
que
o número mínimo esperado de testes está em torno de 2Np^1/2 (Essa observação 
é
devida a M. S. Ralff)

NOTA: Este problema é baseado numa técnica desenvolvida durante a Segunda 
Guerra
Mundial, por R. Dorfman. No exército, Dorfman obteve economia de até 80%. O
aparecimento deste problema despertou uma atenção bastante ampla e conduziu 
a
várias generalizações bem como a novas aplicações industriais e biológicas. 
O
principal aperfeiçoamento consiste em introduzir mais que dois
estágios..

_
MSN Messenger: converse com os seus amigos online.  
http://messenger.msn.com.br

=
Instruções para entrar na lista, sair da lista e usar a lista em
http://www.mat.puc-rio.br/~nicolau/olimp/obm-l.html
=


[obm-l] RE: [obm-l] UM PROBLEMA CLÁSSICO!

2004-09-28 Por tôpico Rogerio Ponce
Olá Jorge e colegas da lista!
Consideremos gotas de água e vinho com o volume V. Portanto, temos 1/V gotas 
em cada vaso.

A cada gota de água que sai e cada gota de vinho que entra, a quantidade de 
água no vaso inferior é diminuída (multiplicada) pelo fator (1-V).

Portanto, ao final do escoamento do vinho, a quantidade de água remanescente 
será igual a
Agua= (1-V) ^ (1/V)  , ou seja,   Agua= e^[ln(1-V) / V ]

E por l´Hopital, quando V-> 0 , Agua ->1/e .
Abraços a todos,
Rogério.
--- from: jorgeluis -
Meus Amigos! Experimentem solucioná-lo sem usar equações diferenciais. Ok!

Um vaso contendo 1 litro de vinho está suspenso sobre outro de igual 
capacidade
cheio de água. Por um orifício no fundo de cada, o vinho escorre sobre o 
vaso
de água e a mistura se esvai na mesma velocidade. Quando o vaso de vinho
estiver vazio, qual é o volume de água no vaso inferior?
_
MSN Messenger: converse com os seus amigos online.  
http://messenger.msn.com.br

=
Instruções para entrar na lista, sair da lista e usar a lista em
http://www.mat.puc-rio.br/~nicolau/olimp/obm-l.html
=


[obm-l] RE: [obm-l] MAIS DIVERSÃO!

2004-09-20 Por tôpico Rogerio Ponce
Olá Jorge e colegas da lista!
Esse problema tem pelo menos 4 soluções:
1) Apague a luz da sala, e provavelmente será possível ver que, quando 
acionados, somente um dos interruptores gera internamente uma pequena 
centelha. Este é o interruptor do sótão.

2) Meça os interruptores com um multímetro.
3) Desconecte um dos fios do interruptor e, após ligar o interruptor, raspe 
o fio no terminal desconectado. O interruptor do sótão gerará centelhas.

4) Ligue um dos interruptores por 3 minutos ( o suficiente para esquentar a 
lâmpada do sótão, caso este seja o interruptor certo) . Então desligue este 
interruptor, ligue um outro qualquer, e corra ao sótão para ver se a lâmpada 
está ligada, ou apagada e quente, ou apagada e fria.

Abraços,
Rogério.

From: jorgeluis
Olá, Pessoal!
Na parede da sala da casa há uma pequena caixa de metal onde estão 
instalados
três interruptores. Um deles acende a lâmpada situada no sótão, cujo acesso 
se
dá por intermédio de uma escada colocada na parede da frente da casa. Os 
outros
dois interruptores podem acender as duas lâmpadas que, aliás estão queimadas 
-,
de duas outras partes da casa. Se do local dos interruptores é totalmente
impossível ver se a luz do sótão está acesa e se somente é permitido ir ao
sótão uma única vez, como descobrir o interruptor que acende a lâmpada lá
instalada?

A propósito, o que ocorrerá se o dono de uma sorveteria usar a mediana ou a 
moda
ao decidir quanto sorvete estocar?

Divirtam-se!
_
MSN Messenger: converse com os seus amigos online.  
http://messenger.msn.com.br

=
Instruções para entrar na lista, sair da lista e usar a lista em
http://www.mat.puc-rio.br/~nicolau/olimp/obm-l.html
=


[obm-l] RE: [obm-l] UM POUCO DE FÍSICA!

2004-09-11 Por tôpico Rogerio Ponce
Boa Claudio!
Esse macaco tinha que ser obra sua...:-)
Só que o macaco estava parado, lembra? ( '...O que acontecerá se o macaco 
resolver subir na corda?' )

E então, como é que ele faz para começar o movimento? Espirrar pra baixo não 
vale!

Não tem jeito, não Claudio: esse teu macaco vai subir tanto quanto o peso do 
outro lado...:-)

Abração,
Rogério.
--
From: Claudio Buffara
Mas e se o macaco subir com velocidade constante? Nesse caso, a forca
resultante sobre ele serah zero, ou seja, a corda exerce sobre ele uma forca
dirigida para cima de magnitude igual ao seu peso. Isso significa que ele
sobe pela corda mas o bloco na outra extremidade da corda ficarah imovel
(desprezando-se a massa da corda e o atrito entre esta e a roldana).
_
MSN Hotmail, o maior webmail do Brasil.  http://www.hotmail.com
=
Instruções para entrar na lista, sair da lista e usar a lista em
http://www.mat.puc-rio.br/~nicolau/olimp/obm-l.html
=


[obm-l] RE: [obm-l] UM POUCO DE FÍSICA!

2004-09-11 Por tôpico Rogerio Ponce
Muito obrigado pelas palavras Auggy!
Mas como você mesmo reparou, nunca qualquer um de nós está sempre 
certo...:-)
Confesso que 'engraxei a meia' com o Coriolis!

Em relação à força aplicada à argola, não basta que ela seja maior que o 
peso: é necessário que ela seja superior à tensão de ruptura dos fios, e que 
passe de zero ao valor máximo num tempo pequeno o suficiente para que a 
movimentação do peso não provoque a ruptura do fio superior. Portanto, ela 
pode ser dita intensa e de rápido crescimento, ou seja, um 'puxão brusco' em 
linguagem coloquial ...:-)

Só pra completar a sequência do fenômeno: a energia transferida ao peso 
durante o esticamento do fio inferior aparecerá logo após a ruptura do mesmo 
sob a forma de oscilação vertical do peso...

Grande abraço,
Rogério.

From: "Qwert Smith"
Grande Rogerio,
vc esta sempre certo e eh um dos meus idolos aki, mas veja os
comentarios abaixo
Olá Jorge e colegas da lista!
Se a roldana for ideal, o macaco vai subir tanto quanto o peso.
Se vc puxar suavemente , quem arrebenta é o fio de cima. Se for um puxão 
brusco, o fio de baixo é quem arrebenta, devido à inércia do corpo pesado.
Aki acho ki faltou defenir puxao brusco... se a forca gerada pelo puxao
for maior ki o peso do corpo entao arrebenta em baixo...se for menor,
mesmo ki sendo um puxao, arrebenta em cima. Pelo menos essa eh
minha modesta opniao.
O sentido do giro da água depende do hemisfério em que vc esteja. A razão 
para o giro é a aceleração de Coriolis, mais forte nos pólos e nula no 
equador.
De acordo com essa pagina Coriolis apenas influencia fenomenos naturais de
grande magnitude.
"The direction of rotation in draining sinks and toilets is NOT determined
by the rotation of the Earth, but by rotation that was introduced earlier
when it was being filled or subsequently being disturbed (say by washing).
The rotation of the Earth does influence the direction of rotation of large
weather systems and large vortices in the oceans, for these are very 
long-lived
phenomena and so allow the very weak Coriolis force to produce a 
significant
effect, with time."

http://www.ems.psu.edu/~fraser/Bad/BadCoriolis.html
[]s,
Auggy
_
MSN Hotmail, o maior webmail do Brasil.  http://www.hotmail.com
=
Instruções para entrar na lista, sair da lista e usar a lista em
http://www.mat.puc-rio.br/~nicolau/olimp/obm-l.html
=


[obm-l] RE: [obm-l] UM POUCO DE FÍSICA!

2004-09-10 Por tôpico Rogerio Ponce
Olá Jorge e colegas da lista!
Se a roldana for ideal, o macaco vai subir tanto quanto o peso.
Se vc puxar suavemente , quem arrebenta é o fio de cima. Se for um puxão 
brusco, o fio de baixo é quem arrebenta, devido à inércia do corpo pesado.

O sentido do giro da água depende do hemisfério em que vc esteja. A razão 
para o giro é a aceleração de Coriolis, mais forte nos pólos e nula no 
equador.


From: jorgeluis
OK! Paulo e demais colegas desta nobre lista! Grato pela retificação em
tempo
real. Ok!
Um macaco está pendurado na extremidade de uma corda que passa por uma
roldana e
é equilibrada por um peso amarrado na outra extremidade. O que acontecerá se
o
macaco resolver subir na corda?
Um corpo pesado está pendurado a uma trave por um fio muito fino. Uma
argola,
bem leve, está pendurada ao corpo pesado por outro fio, tão fino como o
outro.
Se você puxar lentamente a argola para baixo, um dos fios vai arrebentar.
Qual
deles? E se você der um puxão brusco e forte?
A propósito! Por que, quando a água desce pelo ralo, ela gira em sentido
horário
_
MSN Messenger: converse com os seus amigos online.  
http://messenger.msn.com.br

=
Instruções para entrar na lista, sair da lista e usar a lista em
http://www.mat.puc-rio.br/~nicolau/olimp/obm-l.html
=


[obm-l] RE: [obm-l] OFF TOPICO: APENAS PARA OS QUE GOSTAM DE FÍSICA!!!!!!!!!!!!

2004-08-23 Por tôpico Rogerio Ponce
Olá Junior,
não precisa ser físico para perceber que o autor da 'fraude do seculo' não 
entende nada de fotografia.

Todo o espanto relativo à orientação das sombras é facilmente explicado pela 
distância focal utilizada (que implica em distorção perspectiva, etc). O 
mesmo acontece com as diferenças de tamanho da Terra vista da Lua.

Em outros trechos a argumentação é estapafúrdia. Em certo momento, o autor 
considera suspeita a semelhança entre a forma que um astronauta segura uma 
câmera na Lua, e a forma como ele a segura numa instalação da Nasa, dando a 
entender que as fotos do astronauta foram tiradas em sequência. Mas será que 
alguém seguraria a tal câmera de forma diferente? é o mesmo que achar que vc 
seguraria um mesmo telefone de forma diferente conforme o local em que 
estivesse...

Enfim, o site inteiro me parece produto de uma mente delirante, que, como 
sabemos, não é muito difícil de se encontrar...

Abraços,
Rogério.

From: SiarJoes
Gostaria de recolher um pessoal que goste de física para me dizer a 
validades
de alguns seites(endreços a seguir) e desevalidam as teorias de Einsten e
dizem que o homem não foi a lua. Gostaria também que a galera de lista que
conheça matemática avançada desse uma olhada nos cáculos de um físico que 
provou
matemática matente que os cáculos de Einsteme estão errados:

Para os que gostam de matemática click em:
 section1
site sobre a vericidade de Einsten :
www.showdalua.com
E pra terminar teorias de que o homem não foi a lua:
http://www.afraudedoseculo.com.br/

gostaria que os físicos da turma dessem uma olhada nesses sites para mim, e
me digam o veredito.
Abços
Junior
_
MSN Hotmail, o maior webmail do Brasil.  http://www.hotmail.com
=
Instruções para entrar na lista, sair da lista e usar a lista em
http://www.mat.puc-rio.br/~nicolau/olimp/obm-l.html
=


RE: [obm-l] PESQUISA ELEITORAL!

2004-08-20 Por tôpico Rogerio Ponce
Olá Jorge!
Acho que o correto seria "Metade dos entrevistados teve um desempenho abaixo 
da média".

[]'s
Rogério.

From: jorgeluis
Oi, Pessoal!
...
A propósito, o que é absurdo nesta declaração "Metade dos entrevistados, 
tiveram
um desempenho abaixo da média"

Abraços!
_
MSN Messenger: converse com os seus amigos online.  
http://messenger.msn.com.br

=
Instruções para entrar na lista, sair da lista e usar a lista em
http://www.mat.puc-rio.br/~nicolau/olimp/obm-l.html
=


RE: [obm-l] Problema do pica-pau

2004-08-19 Por tôpico Rogerio Ponce
Olá Rafael,
Imagino que você quis dizer que o pica-pau percorreu 20m na vertical (ou 
então ficou faltando esse dado). De qualquer forma o raciocínio seria o 
mesmo.

Como deu 7 voltas de 3m , percorreu 21m na horizontal.
Portanto, o caminho percorrido mede sqtr(20^2 + 21^2) , ou seja, 29m.
[]'s
Rogério.
-
From:Rafael Silva
Olá gente! Queria que vocês resolvessem esse problema para mim (nem que 
pareça besta, é apenas para confirmar meu resultado).

Um pica-pau marca a bicadas seu caminho em uma árvore, realizando seu 
caminho em espiral (como se fosse a hélice de um avião) e começando 20 
metros acima do solo. Sabendo que ele deu 7 voltas em torno da 
circunferência de 3m da árvore, determine a distância total percorrida pelo 
pica-pau.

_
MSN Messenger: converse com os seus amigos online.  
http://messenger.msn.com.br

=
Instruções para entrar na lista, sair da lista e usar a lista em
http://www.mat.puc-rio.br/~nicolau/olimp/obm-l.html
=


Re: [obm-l] ORIGAMI E GEOMETRIA!

2004-08-16 Por tôpico Rogerio Ponce
Olá Dirichlet,
você quis dizer ligue D' em vez de D.
[]'s
Rogério.
-
De: Johann Peter Gustav Lejeune Dirichlet
Assunto: Re: [obm-l] ORIGAMI E GEOMETRIA!
Data: Fri, 13 Aug 2004 11:29:54 -0700
Na verdade, trisseccionar o segmento e mais facil do que voce pensa.
Presta so atençao:traçar paralelas e perpendiculares e facil.
Agora divida o papel em quatro faixas:
A--A'
B--B'
C--C'
D--D'
E--E'
Ligue D com A. O segmento AD sera tiparticionado, e assim, traçando 
paralelas a AE pelos pontos criados, fim!

Boto mais um desafio: obter um poligono regular de sete lados no papel de 
origami. Vai encarar?

_
MSN Messenger: converse com os seus amigos online.  
http://messenger.msn.com.br

=
Instruções para entrar na lista, sair da lista e usar a lista em
http://www.mat.puc-rio.br/~nicolau/olimp/obm-l.html
=


[obm-l] Re: [obm-l] VOLTA ÀS PROBABILIDADES!

2004-08-16 Por tôpico Rogerio Ponce
Olá Jorge,
a probabilidade de 1 cara é
1/2 * 1/2 + 1/2 * 1 = 3/4 , e o aluno estava realmente certo.
Mas a probabilidade das 3 caras COM A MESMA MOEDA é
1/2 * [  (1/2)^3 ]  +  1/2 * [ 1^3 ]  =  9/16
O erro do aluno foi supor que em cada lancamento a moeda seria novamente 
escolhida ao acaso.

Em relacão ao elevador, há divergências...
(eventualmente o seu peso na partida, ao subir, poderá ser maior)
[]'s
Rogério.
---
De: jorgeluis
OK! Artur e valeu Johann pelo esclarecimento, pois desconhecia. Quanto ao
desafio vou queimar um pouco as pestanas!
Um professor de probabilidade propôs a seus alunos o seguinte problema: "São 
dadas duas moedas, uma perfeita (probabilidade de cara igual a 1/2), e outra 
com duas caras. Uma moeda é escolhida ao acaso e lançada 3 vezes. Qual é a 
probabilidade de que sejam obtidas 3 caras?" Um dos alunos, após efetuar 
alguns cálculos, concluiu, corretamente, que se fosse efetuado um único 
lançamento, a probabilidade de se obter uma cara seria igual a 3/4. Como 
foram efetuados três lançamentos independentes, a resposta seria 27/64. O 
que está errado no raciocínio do aluno e qual é a resposta correta do 
problema? Você seria capaz de reformular o problema de modo que o raciocínio 
e a resposta do aluno ficassem corretas?

Vocês sabiam...que ao me pesar dentro de um elevador, a balança acusará 
maior peso quando o elevador estiver parando na descida...

_
MSN Messenger: converse com os seus amigos online.  
http://messenger.msn.com.br

=
Instruções para entrar na lista, sair da lista e usar a lista em
http://www.mat.puc-rio.br/~nicolau/olimp/obm-l.html
=


[obm-l] Re: [obm-l] QUESTÃO MUITO DIFICIL

2004-08-16 Por tôpico Rogerio Ponce
Olá André,
tem certeza de que o enunciado está correto?
A questão tem solucão simples se for LOG(X+2Y) em vez de LOG(X+Y)
[]'s
Rogério.
-
De: andre
Por Favor alguem me ajude a resolver esse problema!
LOG (X + Y) na base 2 + LOG (X - 2Y) na base 3 = 2
X^2 - 4Y^2 = 4
QUANTO VALE X + Y???
ESTOU QUEBRANDO CABEÇA A TEMPOS... ESSA É UMA QUESTÃO DO ITA. ME DISERAM QUE 
TEM QUE TER UMA SACADA PARA RESOLVER.

Obrigado
Atenciosamente, André
_
MSN Messenger: converse com os seus amigos online.  
http://messenger.msn.com.br

=
Instruções para entrar na lista, sair da lista e usar a lista em
http://www.mat.puc-rio.br/~nicolau/olimp/obm-l.html
=


Re: [obm-l] Duvidas

2004-08-05 Por tôpico Rogerio Ponce
Olá Marcos,
a passagem abaixo até funciona, mas o nível de detalhamento destoa das 
demais.
A impressão que dá é que se a=(4+11b)/7 , vc continuaria concluindo que 
b=7c+4 .

[]'s
Rogério.

From: Marcos Paulo
a = (4 + 13b)/7
Como a é suposto inteiro
b = 7c + 4
_
MSN Messenger: converse com os seus amigos online.  
http://messenger.msn.com.br

=
Instruções para entrar na lista, sair da lista e usar a lista em
http://www.mat.puc-rio.br/~nicolau/olimp/obm-l.html
=


Re:[obm-l] Duvidas

2004-08-04 Por tôpico Rogerio Ponce
Ola Fabio,
ha um problema com essa solucao: repare que por este raciocinio, um numero 
que fosse divisivel por N (dando resto zero, portanto) , tambem o seria por 
qualquer multiplo de N.

[]s
Rogerio.

From: "fgb1" Repare que 91 = 13x7
logo o resto será 5x9 = 45
Caso não tenha entendido, pegue um número  com a condição dada e verifique.
Data:Tue, 03 Aug 2004 20:27:03 -0300
Assunto:[obm-l] Duvidas

>
> Como se faz esta questao sem usar congruencias?
>
> Um numero natural ao ser dividido por 7 deixa resto
> 5 e , ao ser dividido por 13 , deixa resto resto 9. O
> resto da divisão desse número por 91 eh igual a:
>
> a0 45 b0 61 c) 65 e)75
>
>
>
> agradeço desde de já.
>
_
MSN Hotmail, o maior webmail do Brasil.  http://www.hotmail.com
=
Instruções para entrar na lista, sair da lista e usar a lista em
http://www.mat.puc-rio.br/~nicolau/olimp/obm-l.html
=


RE: [obm-l] CN 2004

2004-08-04 Por tôpico Rogerio Ponce
Olá Anderson,
lembrando de que o radicando é a quarta potência da soma de dois valores, 
vem:
49 + 20 sqrt(6) = (a+b)^4 = a^4 + 4 (a^3)  b + 6 (a^2) (b^2) + 4 a (b^3) + 
b^4

Um bom chute seria considerar que 20*sqrt(6) corresponde à soma das 
potências ímpares da expansão, ou seja:
20 sqrt(6) = 4 (a^3)  b + 4 a (b^3)
5 sqrt(6) = (a^2 + b^2) * ab
Que tem a solução trivial de a=sqrt(2) e b=sqrt(3)

Testando os valores para 49= a^4 + 6 (a^2) (b^2) + b^4 , verificamos que 
funciona.
Portanto, o radicando equivale à quarta potência de sqrt(2) + sqrt(3)
que nos leva ao resutado de aproximadamente 1,4142 + 1,7321 = 3,1463

[]'s
Rogério

From: Anderson
Aproveitando que esse assunto esta em voga, tentei resolver a questão 
abaixo mas não consegui. Alguém poderia dar alguma dica?

Um aluno resolvendo uma questão de múltipla escolha chegou ao seguinte 
resultado: (49+20(6)^1/2)1/4. No entanto as opçôes  estavam em numeros 
decimais e pedia-se a mais proxima do valor encontrado para resultado, e, 
assim sendo, procurou simplificar este resultado, a fim de melhor estimar a 
resposta. Percebendo que o radicando da rais de indice 4 e quarta potencia 
de uma soma de dois radicais simples, concluiu com maior propriedade que a 
opcao para resposta foi:

Resp: 3,15
Eu fiz a seguinte transformação
(49+20(6)^1/2)1/4 = ((2401)^1/2+(2400)^1/2)1/4
e ai no radicando fiquei com a soma de "dois radicais simples", mas dai 
para diante nao soube como prosseguir.

Obrigado,
Anderson
_
MSN Messenger: converse com os seus amigos online.  
http://messenger.msn.com.br

=
Instruções para entrar na lista, sair da lista e usar a lista em
http://www.mat.puc-rio.br/~nicolau/olimp/obm-l.html
=


[obm-l] RE: [obm-l] Fatorial Ensino Médio

2004-07-30 Por tôpico Rogerio Ponce
Oi Alan,
9 * (2m)! =  (2^m) * m! *  1*3*5*...*(2m+1)
9 * (2m)! =  (2^m) * (1*2*3*...*m)  *  1*3*5*...*(2m+1)
9 * (2m)! =  2*4*6*...*(2m)  *  1*3*5*...*(2m+1)
9 * (2m)! = 1*2*3*...*(2m)*(2m+1)
9 = 2m +1
m=4
[]'s
Rogério.

From: Alan Pellejero Olá amigos da lista,
estou com alguns problemas para resolver esse exercício de fatorial:
Encontre m tal que:
Fat(2m)/[(2^m)*fat(m)*1*3*5...(2m+1) = 1/9  ,
onde fat(m) = m!
Grato,
Alan
_
MSN Messenger: converse com os seus amigos online.  
http://messenger.msn.com.br

=
Instruções para entrar na lista, sair da lista e usar a lista em
http://www.mat.puc-rio.br/~nicolau/olimp/obm-l.html
=


RE: [obm-l] JOGO DO NIM!

2004-07-28 Por tôpico Rogerio Ponce
Olá Jorge e colegas da lista!
Há uma variante mais interessante desse jogo, com a inclusão da seguinte 
regra:

Quando um jogador retira fichas de um monte, ele pode, caso queira, dividir 
o monte restante em dois montes, da forma que julgar mais adequada (1 ficha 
em um monte e as restantes no outro,por exemplo).

Abraços,
Rogério.

From: jorgeluis
OK! Alexandre, Rogério e demais companheiros! Esse tradicional jogo 
proveniente
da China, cuja variante "Resta um" já era jogado por Leibniz em 1716, 
constou
no "banco de problemas" da 6ª Olimpíada Brasileira. Vale a pena ver de 
novo!

Neste jogo, duas pessoas jogam, alternadamente, com uma certa quantidade de
fichas colocadas em vários montes. Em sua vez, o jogador apanha um dos 
montes,
ou quantas fichas quiser em um deles. O que apanhar a última ficha perde. 
Se o
número de fichas em cada monte for expresso no sistema binário, o jogo 
poderá
ser, rapidamente, analisado matematicamente. O jogador que conduza a um 
certo
número de fichas em cada monte poderá ser o vencedor. Como forçar a vitória 
?

Voces sabiam...que uma moeda honesta para efeitos práticos vale 
aproximadamente
1/2 e no sentido matemático é igual a 1/2. Pois, então fiquem 
sabendo...OK!

Abraços e divirtam-se!
_
MSN Hotmail, o maior webmail do Brasil.  http://www.hotmail.com
=
Instruções para entrar na lista, sair da lista e usar a lista em
http://www.mat.puc-rio.br/~nicolau/olimp/obm-l.html
=


RE: [obm-l] Moedas em Cofrinhos

2004-07-28 Por tôpico Rogerio Ponce
Olá Claudio e colegas da lista!
o problema é bonitinho mesmo; é simplesmente o número de soluções não 
negativas de

U1+V1+U2+V2++Um+Vm=n
ou seja, (2m+n-1)! / [(2m-1)! * n!]
( pense em Ui e Vi como sendo, respectivamente, as quantidades de moedas de 
R$1 e R$0,25 no cofrinho "i" )

Grande abraço,
Rogério.


From: "claudio.buffara"
Oi, pessoal:
Um problema bonitinho:
Temos n moedas de R$ 1,00 e n moedas de R$ 0,25.
Moedas de mesma denominacao sao supostas indistinguiveis.
De quantas maneiras podemos escolher n moedas (dentre as 2n que temos) e 
distribui-las por dentre m cofrinhos (cofrinhos podem ficar vazios)?
a) Supondo os cofrinhos numerados de 1 a m.
b) Supondo os cofrinhos indistinguiveis.

[]s,
Claudio.
_
MSN Messenger: converse com os seus amigos online.  
http://messenger.msn.com.br

=
Instruções para entrar na lista, sair da lista e usar a lista em
http://www.mat.puc-rio.br/~nicolau/olimp/obm-l.html
=


[obm-l] RE: [obm-l] UM POUCO DE FÍSICA!

2004-07-26 Por tôpico Rogerio Ponce
Olá Jorge e colegas da lista!
1o. problema:
Puxando-se a corda de x, o peso sobe de x/2.
Como x+x/2 = 1m , x/2=1/3m.
2o. problema:
Lembrando que  s = s0 + v0 * t  + 1/2 * a * t^2 , vem
(4*t + .5*2*t^2) + (2*t + .5*3*t^2) = 1650
2.5*t^2 + 6*t - 1650 = 0
t=[-6 + sqrt(36 + 16500)] / 5
Ou seja,eles se encontram em aproximadamente 24,52 segundos.
(e a resposta do livro está errada!)
3o. problema:
Se o elevador já estiver em velocidade constante, à mesma altura do solo no 
momento da leitura, não importa que tipo de balança você use: a marcação 
será a mesma!

[]'s
Rogério
---
From: jorgeluis
Vocês Sabiam! Que o peso de um corpo é medido com um aparelho chamado
dinamômetro e não com a balança que determina a massa do corpo! Ok!
Numa roldana móvel, ligada por uma corda a outra roldana fixa, está 
pendendo um
certo peso. A extremidade livre da corda está na mesma altura que o peso. 
Se
puxarmos a corda, o peso sobe. Para quanto subirá o peso, se a extremidade 
da
corda se afastar do peso por 1 metro?  Resp: 1/3m

Dois corpos a 1650 metros de distância, movimentam-se um contra o outro a
diferentes velocidades. O primeiro começa com 4 metros por segundo, com
aceleração crescente de 2 metros por segundo. O outro começa com 2 metros 
por
segundo e com aceleração de 3 metros por segundo. Depois de quantos 
segundos se
encontrarâo?  Resp: Após 25 segundos

A propósito, ao me pesar dentro de um elevador, a balança acusará maior 
peso na
subida ou na descida?

_
MSN Messenger: converse com os seus amigos online.  
http://messenger.msn.com.br

=
Instruções para entrar na lista, sair da lista e usar a lista em
http://www.mat.puc-rio.br/~nicolau/olimp/obm-l.html
=


[obm-l] REPETECO

2004-07-26 Por tôpico Rogerio Ponce
Na fazenda de um tio, descobri um barril de cachaça pura, quase vazio e 
preso ao chão.
O barril é dotado de uma micro tela horizontal ao nível de 1L, que impede 
que se retire líquido abaixo da mesma, isto é, este barril nunca tem menos 
que 1L de líquido.

Disponho de uma pipeta e 2 baldes (com capacidade de 2L cada um) , um deles 
vazio, e o outro com 1L de água pura, e pretendo retirar cachaça do barril 
através de acréscimos e retiradas sucessivas de líquido, de forma a ter, ao 
final, 1L de mistura de cachaça e água em um de meus baldes.

Sabendo que cachaça e água misturam-se imediatamente , e de forma homogênea, 
pergunto:
qual o máximo de cachaça que conseguirei retirar do barril?

---
Olá colegas da lista!
Como o Jorge bem lembrou, parece que ninguém fez este...
A questão é descobrir qual a política que maximiza o total de cachaça 
retirada, para então calcular este total.

Reparem que se adicionarmos 1L de água de uma só vez, tiraremos 1L de 
mistura, contendo 1/2 litro de cachaça.

Se adotarmos outra política, adicionando inicialmente 1/2 L de água, 
poderemos retirar 1/2 L de mistura, contendo 1/3L de cachaça.
Adicionando o 1/2L de água restante, tiraremos mais 1/2L de mistura, 
contendo 2/9L de cachaça, e totalizando 5/9L de cachaça retirada.

Qual a solução ótima?
Abraços,
Rogério.
_
MSN Hotmail, o maior webmail do Brasil.  http://www.hotmail.com
=
Instruções para entrar na lista, sair da lista e usar a lista em
http://www.mat.puc-rio.br/~nicolau/olimp/obm-l.html
=


[obm-l] RE: [obm-l] Números complexos

2004-07-26 Por tôpico Rogerio Ponce
Olá Daniele,
pense na representação vetorial de z e w: ambos têm módulo 2, com ângulos de 
45 e 60 graus respectivamente.

Portanto, m vale
|  [(64) + (-48) + (4i)]  /  [(4i) + (-8) + (6) - (2i)]  |  ^  2
ou seja,
| (16+4i) / (-2+2i) | ^ 2  =  (256+16)/(4+4) = 34
Assim, letra "a" é a resposta.
[]'s
Rogério.
---
--> Alguém pode me ajudar nessa questão do ITA?
Considere os números complexos:
z = √2 + i√2  e w = 1 + i√3
Se m = |w^6 + 3z^4 + 4i / z^2 + w^3 + 6 -2i| ^ 2, então m vale:
a) 34
b) 26
c) 16
d) 4
e) 1
--> Outra coisa, alguém sabe onde posso encontrar conceitos sobre  Princípio 
da Indução FFinita ?

Desde já agradeço,
Daniele.
_
MSN Messenger: converse com os seus amigos online.  
http://messenger.msn.com.br

=
Instruções para entrar na lista, sair da lista e usar a lista em
http://www.mat.puc-rio.br/~nicolau/olimp/obm-l.html
=


RE: [obm-l] Outra - Cone Sul 1997

2004-07-24 Por tôpico Rogerio Ponce
Valeu Felipe!
Bem, 10^n significa que temos números com n dígitos.
Para gerarmos todos os números cujos dígitos somam 9(n-1), é como se 
iniciássemos com todos os dígitos iguais a 9, e então somássemos um total de 
-9 unidades a eles, distribuídas de todas as formas possíveis. Portanto, 
queremos o número de solucões não negativas de  A1+..+An = 9 , que vale   
(n+8)! / [ (n-1)! * 9! ]

Para os números cujos dígitos somam 9(n-2), o raciocínio é equivalente: 
queremos distribuir ¨negativamente¨ um total de 18 unidades, com o cuidado 
de eliminar todas as solucões com uma das variáveis maior que 9 (pois não 
poderíamos ter um dígito menor que zero, no número formado). Temos que o 
total de solucões não negativas de A1+...+An=18 vale (n+17)! /  [ (n-1)! * 
18! ] .
E , para o total de solucões com uma das variáveis maior que 9, basta 
imaginar que uma das variáveis já tem o offset de 10, de forma que queremos 
as solucoes nao negativas de A1+...+An = 8 , que vale (n+7)! / [ (n-1)! * 8! 
] .  Esse número deve ser multiplicado por n , pois temos n escolhas para a 
variável com o offset de 10.
Dessa forma, o total para este caso é (n+17)! / [ (n-1)! * 18! ]   -  
n*(n+7)! / [ (n-1)! * 8! ]

Assim, provar que , para n>3, o primeiro caso é menor que o segundo caso, é 
o mesmo que provar que
(n+8)! /9!   +  9n* (n+7)! / 9!   < (n+17)! /  18!

ou
10n+8 < (n+17)*(n+16)*...*(n+8) * 9!/18!
Isso é o mesmo que provar que (chamando Ln de L):
L(10n+8) < L(n+17) + L(n+16) +...+ L(n+8) + L(9!/18!)
Observemos que a derivada dos dois lados é
10/(10n+8) <  1/(n+17) + 1/(n+16) +...+ 1/(n+8) + 0
pois cada um dos termos à direita é maior que 1/(10n+8) , fazendo com que o 
lado direito seja sempre maior que o esquerdo.

Assim, como para n=3 os dois lados valem L(38), e para qualquer n>=3, a 
derivada da direita é sempre maior que a derivada do lado esquerdo, a 
desigualdade é verdadeira.

Abraços,
Rogério.
---
Oi Rogerio,
que tal o enunciado abaixo?

Seja n um número natural, n > 3.
Demonstrar que entre os múltiplos de 9 menores que 10^n há mais números com 
a

soma de seus dígitos igual a 9(n-2) que números com a soma de seus dígitos 
igual a 9(n-1).

 Até mais.   Felipe M.
_
MSN Hotmail, o maior webmail do Brasil.  http://www.hotmail.com
=
Instruções para entrar na lista, sair da lista e usar a lista em
http://www.mat.puc-rio.br/~nicolau/olimp/obm-l.html
=


RE: [obm-l] Outra - Cone Sul 1997

2004-07-22 Por tôpico Rogerio Ponce
Olá Daniel,
tem algum problema com o enunciado:
Para n=4, os múltiplos de 9 menores que 40 são 9,18,27 e 36, nenhum com a 
soma de seus dígitos igual a 18 ou 27.
Portanto, há a MESMA quantidade de números com a soma dos dígitos igual a 
9(n-2) ou 9(n-1) , quando n=4.

[]'s
Rogério.

From: kleinad
Seja n um número natural, n > 3.
Demonstrar que entre os múltiplos de 9 menores que 10n há mais números com 
a
soma de seus dígitos igual a 9(n-2) que números com a soma de seus dígitos
igual a 9(n-1).

[]s,
Daniel
_
MSN Messenger: converse com os seus amigos online.  
http://messenger.msn.com.br

=
Instruções para entrar na lista, sair da lista e usar a lista em
http://www.mat.puc-rio.br/~nicolau/olimp/obm-l.html
=


RE: [obm-l] SORTEIO!

2004-07-22 Por tôpico Rogerio Ponce
Olá Murilo,
não entendi sua conclusão.
Supondo que a composição inicial seja AAE (uma das duas tem que ser sorteada 
no primeiro dia), o mais provável é que no último dia haja 99 bolinhas A e 1 
bolinha E.

Mas o que a intuição já nos diz, pode ser visto de outra forma, assim:
A probabilidade da sequência (AAE)AEAEAA  é igual a 2/3 * 1/4 * 3/5 * 2/6 * 
4/7 * 5/8 , por exemplo.

É fácil ver que qualquer sequência AAE + x bolinhas 'A' + y bolinhas 'E'  
acontece com a probabilidade de:
P1seq(x,y) = [ 2*3*...*(x+1)] * [1*2*...*y] /  [3*4*...*(x+y+2)] = 2 * 
(x+1)! * y! / (x+y+2)!

Assim, considerando todas as sequências possíveis, a probabilidade de 
sortearmos x bolinhas A e y bolinhas E é igual a:
Ptot(x,y) = P1seq(x,y) *  (x+y)! / (x! * y!)
Ptot(x,y) =   2 * (x+1)  / [ (x+y+1) * (x+y+2) ]

Quando houver o total de 100 bolinhas na urna,  x+y vale 97 .
Logo, Ptot(x,y) = (x+1)/(49*99) , que é máximo para x=97 e y=0.
Ou seja, o mais provável é que haja 99 bolinhas da cor que foi sorteada no 
primeiro dia, e uma bolinha da outra cor.

Abraços,
Rogério.

From: "Murilo"
Seja A=vermelha e E=verde
Supunhetemos q A seja sorteada primeiro.
A+E => AA+E
1*AAAE(2/3) = 0.667
1*AAEE(1/3) = 0.333
2*E(2/3)(3/4) = 0.500   4A = 0.500
2*AAAEE(2/3)(1/4) = 0.167   3A = 0.333
2*AAAEE(1/3)(1/2) = 0.167   2A = 0.167
2*AAEEE(1/3)(1/2) = 0.167
3*AE(2/3)(3/4)(4/5) = 0.400   5A = 0.400
3*EE(2/3)(3/4)(1/5) = 0.100   4A = 0.300
3*EE(2/3)(1/4)(3/5) = 0.100   3A = 0.200
3*AAAEEE(2/3)(1/4)(2/5) = 0.067   2A = 0.100
3*EE(1/3)(1/2)(3/5) = 0.100
3*AAAEEE(1/3)(1/2)(2/5) = 0.067
3*AAAEEE(1/3)(1/2)(2/5) = 0.067
3*AA(1/3)(1/2)(3/5) = 0.100
4*AAE(2/3)(3/4)(4/5)(5/6) = 0.333   6A = 0.333
4*AEE(2/3)(3/4)(4/5)(1/6) = 0.067   5A = 0.200
4*AEE(2/3)(3/4)(1/5)(4/6) = 0.067   4A = 0.233
4*EEE(2/3)(3/4)(1/5)(2/6) = 0.033   3A = 0.167
4*EEE(2/3)(1/4)(3/5)(3/6) = 0.033   2A = 0.067
4*EEE(2/3)(1/4)(3/5)(3/6) = 0.033
4*EEE(2/3)(1/4)(2/5)(3/6) = 0.033
4*AAA(2/3)(1/4)(2/5)(3/6) = 0.033
4*AEE(1/3)(1/2)(3/5)(4/6) = 0.067
4*EEE(1/3)(1/2)(3/5)(2/6) = 0.033
4*EEE(1/3)(1/2)(2/5)(3/6) = 0.033
4*AAA(1/3)(1/2)(2/5)(3/6) = 0.033
4*EEE(1/3)(1/2)(2/5)(3/6) = 0.033
4*AAA(1/3)(1/2)(2/5)(3/6) = 0.033
4*AAA(1/3)(1/2)(3/5)(2/6) = 0.067
4*AAE(1/3)(1/2)(3/5)(4/6) = 0.067
Concluimos entao que apos 99 repeticoes o numero de bolas A sera
preferencialmente e aproximadamente 2/3 e bolas E preferencialmente e
aproximadamente 1/3
- Original Message -
From: "Maurizio"
> Uma urna tem inicialmente uma bolinha vermelha e outra verde.
> A cada dia, uma bolinha é sorteada e devolvida à urna junto com mais uma
> outra bolinha da mesma cor.
> Por exemplo, se for sorteada uma bolinha vermelha, a bolinha volta e
> mais uma bolinha vermelha é acrescentada à urna.
>
> É claro que o número de bolinhas no nonagésimo-nono dia é 100.
> Pergunta: Quais resultados você considera mais prováveis para a
> composição da urna, neste dia?
>
_
MSN Messenger: converse com os seus amigos online.  
http://messenger.msn.com.br

=
Instruções para entrar na lista, sair da lista e usar a lista em
http://www.mat.puc-rio.br/~nicolau/olimp/obm-l.html
=


RE: [obm-l] OUTRA RESPOSTA CURIOSA!

2004-07-19 Por tôpico Rogerio Ponce
Olá Jorge,
não recebi a pegadinha da jangada!
Bem, com relação à Terra, fiquei com uma dúvida quase existencial: pendurar 
a Terra em relação a quem?
Acho que um fio de cabelo bastaria, pois ela , por si só, já flutua no 
espaço.

Abraços,
Rogério.

From: jorgeluis
Caro Rogério e demais companheiros, a curiosa resposta dos três círculos 
para a
mesma abertura do compasso foi cunhada por Júlio César de Mello e Souza, 
cujo
pseudônimo de Malba Tahan foi autorizado pelo então presidente Getúlio 
Vargas
com direito a constar na cédula de identidade. Publicou cerca de 56 livros,
porém seu livro mais famoso "O Homem que Calculava" já ultrapassou a 47ª 
edição
e é indicado como livro paradidático em vários países citado na Revista 
Book
Report. Como eu também não entendi esta resposta, vamos adiante com a 
pegadinha
da jangada, que aliás, tá mais para física do que matemática com a soma dos
dois movimentos resultando em 09 transposições. Vejam abaixo outra resposta
curiosa:

Uma esfera de 12,7cm de diâmetro pesa cerca de 1 quilo. Está pendurada num 
fio
ou arame tão delgado que mal dá para sustentá-la. Este arame tem a 
espessura de
0,1mm. A espessura da bola (seu diâmetro) é portanto 1.270 vezes maior que 
a do
arame. Que espessura deveria ter, então, o cabo de aço para nele se 
pendurar a
Terra, que tem um diâmetro 12.700km e pesa 5,56 vezes mais que a água?

NOTA: Considerar o peso da Terra em mais ou menos 53 x 10^20 Toneladas. 
OK!!!

_
MSN Hotmail, o maior webmail do Brasil.  http://www.hotmail.com
=
Instruções para entrar na lista, sair da lista e usar a lista em
http://www.mat.puc-rio.br/~nicolau/olimp/obm-l.html
=


RE: [obm-l] Outra da Escola Naval 2004 - Geometria

2004-07-19 Por tôpico Rogerio Ponce
Olá João,
reveja o enunciado, pois ele está errado.
Abraços,
Rogério.

From: João Vitor
Geometria:
Considere um triangulo ABC, cujos oas lados AB, BC, AC medem 10cm, 25cm,
 10sqrt(2)cm, respectivamente. Seja CH a Altura relativa ao lado AB. Com
centro
no pto médio do lado BC, traça-se uma circunferência que é tangente a CH no
pto T.
O comprimento desta Circunferência é:
A) 25pi/2
B) 25pi/4
C) 21pi/8
D) 5pi/2
E) 5pi/4
Abraços
_
MSN Hotmail, o maior webmail do Brasil.  http://www.hotmail.com
=
Instruções para entrar na lista, sair da lista e usar a lista em
http://www.mat.puc-rio.br/~nicolau/olimp/obm-l.html
=


RE: [obm-l] AS VELAS DO MUCURIPE!

2004-07-17 Por tôpico Rogerio Ponce
Olá Jorge,
não entendi porque apenas 3 círculos...
Bem, vamos à jangada:
15m/(1.5 * 50cm)  = 20
Como a última tranposicão não precisa ser feita (pois a jangada já estará no 
mar), os pescadores farão 19 tranposicões.

[]'s
Rogério.


From: jorgeluis
Bom dia! Meus Amigos! Valeu Rogério, pois não havia resolvido o problema 
das
torneiras e nem o do perfume. Agora, com relação ao problema do compasso, 
posso
traçar sòmente três círculos com a mesma abertura do compasso como também 
não
devo necessáriamente gastar mais tempo nas perguntas que valem mais pontos.
Suponha que você tenha três minutos para responder a um questionário. A
primeira pergunta vale 90 pontos. O primeiro minuto nesta pergunta 
acrescenta
60 pontos a sua pontuação; o segundo minuto acrescenta um ponto. A segunda
pergunta vale 10 pontos. O primeiro minuto adiciona seis pontos; o segundo
minuto, quatro pontos. Utilizando a regra dos pontos marginais por minuto, 
você
deveria gastar um minuto na pergunta 1 e dois minutos na pergunta 2, 
dando-lhe
uma pontuação total de 70 pontos. "Mas, levando as nossas mágoas para as 
águas
fundas do marvida, vento, vela, leva-me daqui..

Para deslocar uma jangada por 15m até o mar, os pescadores utilizam três 
troncos
de carnaúba de 50cm de circunferência. Sabendo-se que a cada volta e meia
simultâneas dos troncos, podem transpor um tronco de trás para frente e 
assim
sucessivamente. Quantas transposições, no mínimo, irão fazer para atingir a
água?  (Clube de Matemática - UFC)

Boa Brisa!
_
MSN Messenger: converse com os seus amigos online.  
http://messenger.msn.com.br

=
Instruções para entrar na lista, sair da lista e usar a lista em
http://www.mat.puc-rio.br/~nicolau/olimp/obm-l.html
=


RE: [obm-l] Geometria plana

2004-07-16 Por tôpico Rogerio Ponce
Olá Guilherme,
PA+PC >= AC >= PB
[]'s
Rogério.

From: "Guilherme"
Olá, pessoal!
Aqui vai um problema proposto pela Universidade de Wisconsin. O concurso
já acabou, em 10 de março de 2004, mas fiquei curioso para saber como
resolvê-lo:
ABCD é um quadrado e P é um ponto interior a ele. Mostre que as
distâncias PA, PB e PC satisfazem a inequação PA + PC  >= PB  (maior ou
igual). (Na verdade, é irrelevante o fato de P ser interior ao quadrado.
A inequação é válida para todos os pontos P no plano).
Agradeço a ajuda.
Um grande abraço,
Guilherme Marques.
_
MSN Messenger: converse com os seus amigos online.  
http://messenger.msn.com.br

=
Instruções para entrar na lista, sair da lista e usar a lista em
http://www.mat.puc-rio.br/~nicolau/olimp/obm-l.html
=


[obm-l] RE: [obm-l] PROBLEMAS CLÁSSICOS!

2004-07-16 Por tôpico Rogerio Ponce
Olá Jorge e colegas da lista!
Acho que estes 2 problemas foram esquecidos.

From: jorgeluis
Oi, Pessoal!
Uma linha de bondes liga duas estações distantes de 9km; de cada estação, 
saem
carros de 3 em 3 minutos, andando com mesma velocidade uniforme nos dois
sentidos. Um viajante percorre a pé o mesmo caminho com velocidade 
uniforme. Vê
um carro chegar e outro sair, quando parte de uma estação e quando chega à
outra estação. Na viagem, encontra 17 bondes indo no mesmo sentido que ele 
e 41
no sentido contrário, não contanto os carros da saída e da chegada. 
Calcular a
velocidade do viajante.
---
Solução:
Tempos de percurso entre estações, do viajante e de um bonde: "Tv" e "Tb"
Quando o viajante parte, existem (Tb/3 - 1) bondes pelo caminho, e durante a 
viagem, partem mais Tv/3 bondes de

cada uma das estações.
Portanto 41 = Tb/3 - 1 + Tv/3
Supondo o viajante mais rápido que um bonde:   (Tb/3 - 1) - (Tv/3) = 17
Se o viajante for mais lento:  (Tv/3 - 1) - Tb/3 = 17
Assim, Tv=36 ou Tv=90, isto é, o viajante poderia estar a 9*60/36= 15km/h 
(com os bondes a 6km/h) ou a 9*60/90= 6km/h (com os bondes a 15km/h).

---

Duas torneiras alimentam um reservatório. A 1ª dá, por minuto, 2 litros 
mais que
a 2ª. Abre-se a 1ª durante a metade do tempo que a 2ª levaria para encher o
reservatório. Fecham-na então e abre-se a 2ª que acaba de encher o tanque. 
Se
as duas tivessem sido abertas juntas, o tanque encher-se-ia 18h 47min mais 
cedo
e a 1ª torneira só teria fornecido os 5/6 do que realmente forneceu. 
Quantos
litros deu cada torneira por minuto? Qual é a capacidade do reservatório? 
Que
tempo levariam juntas? Gostaria da resolução desta! Abraços!

---
Solução:
vazão das duas torneiras: "a" e "b"
volume do reservatório: "v"
tempo de abertura das torneiras:  "Ta" e "Tb"
(1) a = b + 2
(2) Ta = (v/b)/2
(3) Tb = (v - a*Ta) / b
(4) v/(a+b) = Ta + Tb - 18h47min
(5) a * v/(a+b) = 5/6 * a * (v/b)/2
QUanto vale a,b,v, v/(a+b) ?
(1) e (5) vem:  1/(2b+2) = 5/12b  -> 12b = 10b+10 -> b=5 , a=7
(4)v/12 = v/10 + (v-7v/10)/5  - 1127  ->  v * (1/12 - 1/10 - 3/50) = 
-1127 ou v*(25-30-18)/300 = -1127
v=14700
v/(a+b) = 1225 min = 20h 25min

Abraços,
Rogério
_
MSN Messenger: converse com os seus amigos online.  
http://messenger.msn.com.br

=
Instruções para entrar na lista, sair da lista e usar a lista em
http://www.mat.puc-rio.br/~nicolau/olimp/obm-l.html
=


RE: [obm-l] log-poli

2004-07-15 Por tôpico Rogerio Ponce
Olá Junior,
1/bc + 1/ac + 1/ab = (a+b+c)/abc = (30/2) / (3/2) = 10
E log de 10 na base .1 é -1.
[]'s
Rogério

From:
Questão de logarítimo com polinômio
anexo
abraços
Junior
_
MSN Messenger: converse com os seus amigos online.  
http://messenger.msn.com.br

=
Instruções para entrar na lista, sair da lista e usar a lista em
http://www.mat.puc-rio.br/~nicolau/olimp/obm-l.html
=


RE: [obm-l] O JOGO DOS 15!

2004-07-15 Por tôpico Rogerio Ponce
Também seria possível apoiar a ponta seca do compasso sobre uma caixa de 
fósforos colocada sobre o papel.
O raio da circunferência tracada iria diminuindo à medida que a altura da 
caixa fosse aumentando.
Com este método, vc tracaria uma infinidade de circunferências de raios 
inferiores à abertura do compasso.

Abracos,
Rogerio.

From: "Rogerio Ponce" Olá Jorge e colegas da lista!
Bem, vc pode tracar uma infinidade de circunferências com um compasso 
'fixo' .
Além da circunferência trivial para aquela abertura, vc poderia fazer o 
seguinte:

1o. método:
Use a base de um copo, e , com a ponta de grafite do compasso, trace a sua 
circunferência. Como pode existir uma infinidade de bases diferentes, vc 
poderia tracar uma infinidade de circunferências.

2o. método:
Coloque a ponta seca do compasso sobre cada ponto da circunferência 
trivial, e trace outra circunferência. Após percorrer toda a circunferência 
original, o seu desenho externo será uma circunferência com o dobro do 
raio. Como esse processo pode ser aplicado à cada circunferência, vc 
poderia obter todas as circunferências de raios 1R, 2R,3R etc.

Acordei de bom humor.
Bom dia a todos!
Rogério.


From: jorgeluis
Meus Colegas! O jogo dos 15 consiste numa caixa quadrada e fina de madeira 
ou
metal, que contém 15 pequenos blocos quadrados numerados de 1 a 15. Há, na
realidade, espaço para 16 blocos na caixa, de modo que os 15 podem ser 
movidos
e trocar de lugar. O número de posições concebíveis é 
16!=20.922.789.888.000.
Um problema consiste em arrumar os blocos de uma determinada maneira, 
partindo
de uma posição inicial dada, que é, frequentemente, a posição normal. Dois
matemáticos americanos provaram que, de qualquer posição inicial dada, 
apenas
metade de todas as posições concebíveis pode ser realmente conseguida.
Portanto, há sempre aproximadamente dez trilhões de posições que o 
possuidor de
um jogo dos 15 pode atingir e dez trilhões que ele não pode. O espaço 
vazio
deve mover-se em um número par de espaços. Se, partindo da posição normal,
pode-se conseguir a posição desejada de acordo com aquele requisito, é uma
posição possível; de outro modo, é impossível. Toda situação em que um 
número
preceder outro menor que ele é chamada de inversão.

A propósito, quantos círculos diferentes posso traçar com a mesma abertura 
do
compasso?

_
MSN Messenger: converse com os seus amigos online.  
http://messenger.msn.com.br

=
Instruções para entrar na lista, sair da lista e usar a lista em
http://www.mat.puc-rio.br/~nicolau/olimp/obm-l.html
=
_
MSN Messenger: converse com os seus amigos online.  
http://messenger.msn.com.br

=
Instruções para entrar na lista, sair da lista e usar a lista em
http://www.mat.puc-rio.br/~nicolau/olimp/obm-l.html
=


[obm-l] RE: [obm-l] MAIS DÚVIDAS!

2004-07-14 Por tôpico Rogerio Ponce
É VERDADE, RAFAEL !
[]'s
Rogério.

From: Rafael Ando
Rogerio Ponce wrote:
Olá Jorge e colegas da lista!
1o. problema
Como a interseção da superfície esférica com o plano é uma circunferência, 
um dos pontos tem que ser, necessariamente o ponto externo ao plano. E 
cada grupo de 3 pontos do plano definirá , com o ponto externo, uma nova 
esfera. Como temos 19 pontos no plano, podemos definir ao todo
19! / (16! * 3!)  = 969 superfícies esféricas

2o. problema
Ela acha que tanto faz uma coisa ou outra.
Portanto o valor esperado da aposta é $20.
Como a chance dos $2 é 1/3 , e a do perfume é 2/3 , podemos escrever que
$20 = $2 * 1/3 + P * 2/3
Logo, P = $29
[]'s
Rogério.

From: jorgeluis
OK! Guilherme! Esse tiro de misericórdia foi certeiro e em tempo real! 
Grato!!

Num plano são dados 19 pontos entre os quais não se encontram 3 alinhados 
e nem
4 situados numa mesma circunferência. Fora do plano é dado mais um ponto.
Quantas superfícies esféricas existem cada uma passando por 4 dos pontos 
dados?

NOTA: Tenho impressão que este já saiu na lista, mas como não 
encontrei.?


A Sra. Black acha que é uma questão aleatória aceitar $20 em dinheiro ou 
fazer
uma aposta, extraindo uma bola de uma urna com 15 bolas brancas e 45 
vermelhas,
onde ela receberá $2 se a bola extraída for branca ou um perfume se a 
bola for
vermelha. Qual valor ela atribui ao perfume?


Um abraço à todos!

na verdade, a chance dos $2 eh 1/4 e o perfume 3/4... pelo mesmo raciocínio 
o perfume vale $26.
_
MSN Messenger: converse com os seus amigos online.  
http://messenger.msn.com.br

=
Instruções para entrar na lista, sair da lista e usar a lista em
http://www.mat.puc-rio.br/~nicolau/olimp/obm-l.html
=


[obm-l] RE: [obm-l] MAIS DÚVIDAS!

2004-07-14 Por tôpico Rogerio Ponce
Olá Jorge e colegas da lista!
1o. problema
Como a interseção da superfície esférica com o plano é uma circunferência, 
um dos pontos tem que ser, necessariamente o ponto externo ao plano. E cada 
grupo de 3 pontos do plano definirá , com o ponto externo, uma nova esfera. 
Como temos 19 pontos no plano, podemos definir ao todo
19! / (16! * 3!)  = 969 superfícies esféricas

2o. problema
Ela acha que tanto faz uma coisa ou outra.
Portanto o valor esperado da aposta é $20.
Como a chance dos $2 é 1/3 , e a do perfume é 2/3 , podemos escrever que
$20 = $2 * 1/3 + P * 2/3
Logo, P = $29
[]'s
Rogério.

From: jorgeluis
OK! Guilherme! Esse tiro de misericórdia foi certeiro e em tempo real! 
Grato!!

Num plano são dados 19 pontos entre os quais não se encontram 3 alinhados e 
nem
4 situados numa mesma circunferência. Fora do plano é dado mais um ponto.
Quantas superfícies esféricas existem cada uma passando por 4 dos pontos 
dados?

NOTA: Tenho impressão que este já saiu na lista, mas como não 
encontrei.?


A Sra. Black acha que é uma questão aleatória aceitar $20 em dinheiro ou 
fazer
uma aposta, extraindo uma bola de uma urna com 15 bolas brancas e 45 
vermelhas,
onde ela receberá $2 se a bola extraída for branca ou um perfume se a bola 
for
vermelha. Qual valor ela atribui ao perfume?


Um abraço à todos!
_
MSN Messenger: converse com os seus amigos online.  
http://messenger.msn.com.br

=
Instruções para entrar na lista, sair da lista e usar a lista em
http://www.mat.puc-rio.br/~nicolau/olimp/obm-l.html
=


RE: [obm-l] O PROBLEMA DOS PONTOS!

2004-07-14 Por tôpico Rogerio Ponce
Olá Jorge!
B só ganha se vencer 3 rodadas seguidas, caso contrário perde. Portanto, a 
chance de B ganhar é de 1/2^3 = 1/8 . Assim, a chance de A ganhar é de 1 - 
1/8 = 7/8 .
Dessa forma, A deve receber 7 vezes mais que B , ou seja,  7/8 das apostas 
vão para A , e 1/8 vai para B.
[]'s
Rogério


From: jorgeluis
A e B estão empenhados em um honesto jogo de balla. Eles concordam em 
continuar
até que um deles vença seis rodadas. O jogo realmente termina quando A 
venceu
cinco, e B, três rodadas. Como devem ser divididas as apostas?

Vale salientar que o enigma de Paccioli é mais significativo do que 
aparenta,
pois marcou o início da análise sistemática da probabilidade e nos leva ao
limiar da quantificação do risco. A solução do problema dos pontos começa 
pelo
reconhecimento de que o jogador que está vencendo quando o jogo é 
interrompido
teria maiores probabilidades de vitória se o jogo prosseguisse. Mas quão
maiores são essas chances do jogador que está vencendo? Quão pequenas são 
as
chances do jogador que está perdendo?

Pascal e Fermat atacaram o problema de diferentes perspectivas. Fermat 
voltou-se
para a álgebra pura. Pascal foi mais inovador: usou um formato geométrico 
para
esclarecer a estrutura algébrica subjacente. O conceito matemático básico 
por
trás dessa álgebra geométrica foi reconhecido muito antes de Fermat e 
Pascal a
adotarem. Omar Khayyam estudara-o cerca de 450 anos antes. Em 1303, um
matemático chinês chamado Chu Shih-chieh abordou o problema mediante um
dispositivo que denominou o "Espelho Precioso dos Quatro Elementos" que 
passou
a ser conhecido como o "Triângulo de Pascal".

A propósito, qual a diferença entre uma moeda honesta para efeitos práticos 
e em
sentido matemático?  Abraços!!!


__
WebMail UNIFOR - http://www.unifor.br.
=
Instruções para entrar na lista, sair da lista e usar a lista em
http://www.mat.puc-rio.br/~nicolau/olimp/obm-l.html
=
_
MSN Hotmail, o maior webmail do Brasil.  http://www.hotmail.com
=
Instruções para entrar na lista, sair da lista e usar a lista em
http://www.mat.puc-rio.br/~nicolau/olimp/obm-l.html
=


Re: [obm-l] OBM 2003- Alguem fez esse?

2004-07-13 Por tôpico Rogerio Ponce
Olá Domingos,
sua solução está perfeita!
Eu havia imaginado uma solução um pouquinho diferente:
Se houver apenas um meio de transporte para as N cidades, a inserção de X 
pode ocorrer em qualquer lugar, e a solução é trivial.

Com 2 meios de transporte, suponhamos A B C cidades consecutivas, com a 
transição em B. Sem perda de generalidade, AB pode ser considerada ferrovia, 
e BC pode ser rodovia.
Se XB for ferrovia, faca a inclusão de X entre B e C, obtendo o circuito A B 
X C.
Se XB for rodovia, faca a inclusão de X entre A e B, obtendo o circuito A X 
B C.

Abraços,
Rogério.

Olá pessoal, esqueceram de fazer este problema...
Há N cidades em Tumbólia. Cada duas cidades desse país são
ligadas por uma rodovia ou uma ferrovia, não existindo nenhum
par de cidades ligadas por ambos os meios.
Um turista deseja viajar por toda a Tumbólia, visitando cada cidade
exatamente uma vez, e retornar a cidade onde ele começou sua jornada.
Prove que é possível escolher a ordem na qual as cidades serão visitadas
de modo que o turista mude o meio de transporte no máximo uma vez.
Esse é um problema de grafos.
Veja que para N = 2 o problema é trivial.
Vamos mostrar que para N + 1 também conseguimos resolver o problema.
Agora Tumbólia tem N + 1 cidades e já montamos um circuito (um itinerário 
onde visitamos cada cidade uma única vez) de tamanho N onde trocamos de 
transporte no máximo 1 vez. Suponha que X seja a cidade que ficou de fora 
deste circuito.

Se o circuito formado só possui um meio de transporte, podemos adicionar X 
ao circuito e trocar de meio de transporte no máximo 1 vez (faça um 
desenho...).

Se o circuito formado já troca de transporte ao passar pela cidade Y 
teremos cuidados extras. Podemos assumir (sem perda de generalidade) que o 
circuito de N cidades começa em Y e chega até Z através de rodovia e de Z 
até Y utiliza ferrovia.

Se X se liga a Y através de ferrovia, insira X logo após Y no circuito e 
veja que isso dá uma solução.
Analogamente, se X se liga a Z através de ferrovia, então coloque X 
imediatamente antes de Z no circuito.

Sobrou apenas o caso em que X se liga a Y e Z através de rodovias.
Neste caso forme um circuito começando Z, passando por X, depois Y, depois 
siga o circuito original a partir de Y até encontrar o predecessor de Z, Z' 
(no circuito original), vá de Z' até o antecessor de Y, Y' (também no 
circuito original) e depois siga até Z pela direção contrária ao circuito 
original.

Por texto fica confuso, mas se você desenhar, vai ver que essa construção 
funciona.

[ ]'s
_
MSN Messenger: converse com os seus amigos online.  
http://messenger.msn.com.br

=
Instruções para entrar na lista, sair da lista e usar a lista em
http://www.mat.puc-rio.br/~nicolau/olimp/obm-l.html
=


[obm-l] RE: [obm-l] questão chatíssima

2004-07-13 Por tôpico Rogerio Ponce
Olá Junior,
vamos por ítens:
(1)
Coordenadas cartesianas de A:
x=40*cos(120)=-20
y=40*sin(120)=20*sqrt(3)
Coordenadas cartesianas de B:
x=30*cos(330)=15*sqrt(3)
y=30*sin(330)=-15
Distância entre A e B
= sqrt[  (15*sqrt(3) - -20)^2  +  (-15 -20*sqrt(3))^2 ] =
= sqrt(225*3+400+400*3+225+1200*sqrt(3))
= 67.6 , portanto, MENOR que 69.
(2)
Se os navios andarem em linha reta para onde?!
Para onde cada uma das proas está apontada?
Nem o Grael responde esse ítem...
(3)
Se B estiver a 40km/h, em 10min terá percorrido 40/6 km, ou seja, um arco de 
4/18 radianos, ou seja, 40/pi graus.

Abraços,
Rogério.

From: SiarJoes
quem puder ajudar
anexo
abços
Junior
<< dificil.GIF >>
_
MSN Hotmail, o maior webmail do Brasil.  http://www.hotmail.com
=
Instruções para entrar na lista, sair da lista e usar a lista em
http://www.mat.puc-rio.br/~nicolau/olimp/obm-l.html
=


[obm-l] OBM 2003- Alguem fez esse?

2004-07-13 Por tôpico Rogerio Ponce
Olá pessoal, esqueceram de fazer este problema...
Há N cidades em Tumbólia. Cada duas cidades desse país são
ligadas por uma rodovia ou uma ferrovia, não existindo nenhum
par de cidades ligadas por ambos os meios.
Um turista deseja viajar por toda a Tumbólia, visitando cada cidade
exatamente uma vez, e retornar a cidade onde ele começou sua jornada.
Prove que é possível escolher a ordem na qual as cidades serão visitadas
de modo que o turista mude o meio de transporte no máximo uma vez.
[]'s
Rogério.
_
MSN Messenger: converse com os seus amigos online.  
http://messenger.msn.com.br

=
Instruções para entrar na lista, sair da lista e usar a lista em
http://www.mat.puc-rio.br/~nicolau/olimp/obm-l.html
=


[obm-l] RE: [obm-l] RETIFICAÇÕES!

2004-07-13 Por tôpico Rogerio Ponce
Olá Jorge!
acho que nesses dois vc deu uma escorregada...:-)
1o. problema:
Vejamos o que vc enunciou anteriormente:
As cinco finalistas de um concurso de beleza têm os olhos verdes
ou negros. O apresentador do desfile anunciou que todas as candidatas
com olhos negros têm uma pequena tatuagem na nuca. Elas entraram
no palco com as luzes apagadas e, quando os refletores se acenderam,
duas estavam de frente e três de costas.
Quantas garotas, no mínimo, devem dar meia volta para que a platéia
descubra se o apresentador disse a verdade?
E se houver 2 garotas de olhos negros de frente, e 3 de olhos verdes de 
costas?
Dizem que nesse caso, todas as 5 precisariam dar meia volta...

Então, de onde vc foi tirar que basta apenas uma garota se voltar ?!
Conforme eu disse antes, será necessário que se virem as garotas de costas 
sem tatuagem na nuca, assim como as de frente que possuam olhos negros.

---
2o. problema:
Comentando a sua resposta, vem:
Agora, quanto ao café com leite, o esfriamento é
proporcional à diferença de temperatura entre o corpo
que vai ser esfriado e o meio ambiente.
   Está correto.

Se você começar misturando o leite ao café, a diferença
entre a temperatura deste e a do meio ambiente diminui,
   Está correto.

... e assim ele demorará mais para
chegar ao ponto desejado.
   Está errado.
O café demoraria mais para ter a mesma queda de temperatura. Mas vc se 
esquece de que agora, a queda necessária para chegar à mesma temperatura 
final é MENOR, pois foi misturado com leite frio!
Dessa forma, o tempo gasto continua o mesmo.


Ao contrário, se você deixar o café muito quente
exposto à temperatura ambiente, a diferença maior entre ambos fará com que
aquele esfrie mais depressa.
Melhor dizer que a queda de temperatura do café no mesmo tempo será maior, 
porém o café estará começando a esfriar a partir de uma temperatura mais 
alta. Ao final dos 5 minutos, a temperatura será a mesma.

Conforme eu disse antes, TANTO FAZ  misturar antes ou depois !
Dê uma olhadinha nas equações que escrevi que ficará mais fácil de entender.
Abraços,
Rogério.
--

From: jorgeluis
Ok! Rogério e demais amigos! Quanto ao concurso de beleza basta sòmente uma
única garota dar meia volta. O apresentador disse que, se uma candidata tem
olhos negros, então ela tem uma tatuagem na nuca. Esta afirmação estaria 
errada
se, e somente se, houvesse uma candidata de olhos negros que não tivesse 
uma
tatuagem na nuca. A moça de olhos verdes não precisa voltar-se, pois nada 
foi
dito à respeito das moças de olhos verdes. Basta verificar apenas se a moça 
de
olhos negros tem uma tatuagem na nuca. Se tiver, o apresentador disse a
verdade; se não tiver, mentiu.

Agora, quanto ao café com leite, o esfriamento é
proporcional à diferença de temperatura entre o corpo que vai ser esfriado 
e o
meio ambiente. Se você começar misturando o leite ao café, a diferença 
entre a
temperatura deste e a do meio ambiente diminui, e assim ele demorará mais 
para
chegar ao ponto desejado. Ao contrário, se você deixar o café muito quente
exposto à temperatura ambiente, a diferença maior entre ambos fará com que
aquele esfrie mais depressa. Ao juntar-lhe o leite, finalmente, ele chegará 
ao
ponto certo. Ok!

A propósito, porque quando multiplicamos em uma calculadora x.1/x não 
obtemos um
resultado igual a 1, mas uma fração como 0,?  (É A NOVA..)

_
MSN Messenger: converse com os seus amigos online.  
http://messenger.msn.com.br

=
Instruções para entrar na lista, sair da lista e usar a lista em
http://www.mat.puc-rio.br/~nicolau/olimp/obm-l.html
=


[obm-l] RE: [obm-l] RECREAÇÃO! (Problema dos Avioes)

2004-07-12 Por tôpico Rogerio Ponce
Vamos lá:
O 3 aviões A,B e C partem juntos na mesma direção, de tanque cheio, sendo 
que o avião C dá a volta ao mundo, e é sempre reabastecido por B, que por 
sua vez é sempre reabastecido pela base, ou por A, que sempre é reabastecido 
pela base.

Avião C: voa PI/2, recebe combustível de B equivalente a mais PI/2, voa mais 
PI, recebe combustível de B equivalente a mais PI/2, e completa a volta ao 
mundo.

Avião B: voa até PI/4, recebe combustível de A equivalente a mais PI/4, voa 
mais PI/4 e abastece C com o equivalente a PI/4, faz meia volta, e retrocede 
até PI/4 , onde recebe combustível de A para mais PI/4, que é o suficiente 
para chegar à base. Ao chegar à base, repete o mesmo processo pelo outro 
lado, voando até -PI/2, abastecendo C e retornando à base, sendo 
reabastecido por A (com combustível para mais PI/4) em -PI/4 , na ida e na 
volta.

Avião A: voa PI/4, abastece B com PI/4, retorna à base, enchendo o tanque, 
volta até PI/4, abastece B com PI/4, retorna à base, enchendo o tanque, e 
repete o processo pelo outro lado, voando até -PI/4, abastecendo B com PI/4, 
etc,etc.

Abraços,
Rogério.


From: "Qwert Smith"
Eu gostaria de ver a solucao com 3 avioes.  Eu so consigo imganiar a volta
com o uso de 6 avioes.  Se imaginarmos que o reabastecimento de um
aviao eh instantaneo e ele pode ser reusado ai a conta cai pra 4 avios, mas
nao 3.
...
From: "Rogerio Ponce"
Olá Jorge,
com 3 aviões já é possível dar a volta ao mundo.
From: jorgeluis
Qual é o menor número de aviões necessários par que um deles dê a volta 
ao redor
do mundo, pelo equador, sabendo que: todos saem da mesma base, que é a 
única
fonte de combustível e pela qual passa o equador; cada um pode levar
combustível suficiente para meia volta; é possível passar combustível de 
um
avião para outro, em pleno vôo, sem perder tempo? Considerando que todos 
os
aviões têm a mesma média de velocidade e consumo de combustível, que só 
podem
aterrisar na ilha e regressam em perfeitas condições.
_
MSN Messenger: converse com os seus amigos online.  
http://messenger.msn.com.br

=
Instruções para entrar na lista, sair da lista e usar a lista em
http://www.mat.puc-rio.br/~nicolau/olimp/obm-l.html
=


[obm-l] RE: [obm-l] RECREAÇÃO!

2004-07-12 Por tôpico Rogerio Ponce
Olá Daniel,
essa equação traduz a variação exponencial (ao longo do tempo) da 
temperatura de um corpo quando imerso em um ambiente com outra temperatura.

O 'alfa' tem a ver com as características de troca de energia do corpo com o 
ambiente, e 't' é o tempo decorrido desde o início da imersão.

Assim, para t=0, por exemplo, o corpo se encontra em sua temperatura inicial 
( 'Tinicial' ), e , à medida que 't' cresce, a temperatura do corpo ( 
'Tfinal' ) se aproxima assintoticamente da temperatura do ambiente ( 
'Tambiente' ).

Abraços,
Rogério.

From: "Daniel Regufe"
Ola Rogerio ...
[...Tfinal =  ( Tinicial - Tambiente )  *  e^ ( - alfa * t )   +  
Tambiente...]

Q equação eh essa ... pode me explicar melhor?
[]`s
Regufe
_
MSN Hotmail, o maior webmail do Brasil.  http://www.hotmail.com
=
Instruções para entrar na lista, sair da lista e usar a lista em
http://www.mat.puc-rio.br/~nicolau/olimp/obm-l.html
=


RE: [obm-l] O JOGO DOS 15!

2004-07-12 Por tôpico Rogerio Ponce
Olá Jorge e colegas da lista!
Bem, vc pode tracar uma infinidade de circunferências com um compasso 'fixo' 
.
Além da circunferência trivial para aquela abertura, vc poderia fazer o 
seguinte:

1o. método:
Use a base de um copo, e , com a ponta de grafite do compasso, trace a sua 
circunferência. Como pode existir uma infinidade de bases diferentes, vc 
poderia tracar uma infinidade de circunferências.

2o. método:
Coloque a ponta seca do compasso sobre cada ponto da circunferência trivial, 
e trace outra circunferência. Após percorrer toda a circunferência original, 
o seu desenho externo será uma circunferência com o dobro do raio. Como esse 
processo pode ser aplicado à cada circunferência, vc poderia obter todas as 
circunferências de raios 1R, 2R,3R etc.

Acordei de bom humor.
Bom dia a todos!
Rogério.


From: jorgeluis
Meus Colegas! O jogo dos 15 consiste numa caixa quadrada e fina de madeira 
ou
metal, que contém 15 pequenos blocos quadrados numerados de 1 a 15. Há, na
realidade, espaço para 16 blocos na caixa, de modo que os 15 podem ser 
movidos
e trocar de lugar. O número de posições concebíveis é 
16!=20.922.789.888.000.
Um problema consiste em arrumar os blocos de uma determinada maneira, 
partindo
de uma posição inicial dada, que é, frequentemente, a posição normal. Dois
matemáticos americanos provaram que, de qualquer posição inicial dada, 
apenas
metade de todas as posições concebíveis pode ser realmente conseguida.
Portanto, há sempre aproximadamente dez trilhões de posições que o 
possuidor de
um jogo dos 15 pode atingir e dez trilhões que ele não pode. O espaço vazio
deve mover-se em um número par de espaços. Se, partindo da posição normal,
pode-se conseguir a posição desejada de acordo com aquele requisito, é uma
posição possível; de outro modo, é impossível. Toda situação em que um 
número
preceder outro menor que ele é chamada de inversão.

A propósito, quantos círculos diferentes posso traçar com a mesma abertura 
do
compasso?

_
MSN Messenger: converse com os seus amigos online.  
http://messenger.msn.com.br

=
Instruções para entrar na lista, sair da lista e usar a lista em
http://www.mat.puc-rio.br/~nicolau/olimp/obm-l.html
=


RE: [obm-l] CADEIAS DE MARKOV!

2004-07-11 Por tôpico Rogerio Ponce
Olá Jorge e colegas da lista!
1o. problema:
Partindo de um Ford, em 1/3 das vezes ele fica com um Ford , em outro 1/3 
das vezes ele se divide em um Dodge, um Chevrolet e um Ford , e em outro 1/3 
das vezes ele se divide em um Chevrolet e um Ford. Ou seja, ele fica com um 
Ford em ( 1/3 ) + ( 1/3 * 1/3 ) + ( 1/3 * 1/2 ) = 11/18 do tempo.

2o. problema:
À noite, a velocidade tangencial da superfície da Terra em relacão ao seu 
eixo de rotacão (sobre si mesma) se soma à velocidade tangencial da Terra em 
relacão ao Sol.
Portanto, nosso movimento é mais rápido à noite.
O ponto mais veloz acontece na 'meia-noite solar' , isto é, quando o 
meridiano oposto a você está exatamente em frente ao sol  (meio-dia solar) .

Abracos,
Rogério.

From: jorgeluis
Olá, Pessoal!
Todo ano um homem troca o seu carro por um carro novo. Se ele tem um Dodge,
troca-o por um Chevrolet. Se tem um Chevrolet, troca-o por um Ford. No 
entanto,
se tem um Ford, a probabilidade de que ele o troque por um novo Ford é a 
mesma
de que por um Dodge ou um Chevrolet. A longo prazo, durante que parte do 
tempo
ele terá um Ford?

A propósito, quando você se move mais depressa em relação ao Sol, de dia ou 
de
noite?

A todos, uma bela tarde de domingo.
_
MSN Hotmail, o maior webmail do Brasil.  http://www.hotmail.com
=
Instruções para entrar na lista, sair da lista e usar a lista em
http://www.mat.puc-rio.br/~nicolau/olimp/obm-l.html
=


<    1   2   3   4   5   6   7   >